Vous êtes sur la page 1sur 108

Print ACE11A

5/25/14, 3:36 PM

ACE 11A

QUESTION

1.

A patient undergoing lumbar spine fusion surgery is monitored by measuring lower extremity
somatosensory evoked potentials (SSEPs). Which of the following is MOST likely to cause a
50% increase from baseline in the SSEP latency?
A. Anterior spinal artery ischemia
B. Compression of the anterior column of the spinal cord
C. Changing from an inhaled anesthetic to a total intravenous anesthetic
D. Compression of the dorsal column of the spinal cord

Comments on the incorrect answers


B.

Anterior spinal cord compression may not have any effect on SSEPs.

C.

Changing to a total intravenous anesthetic would most likely decrease SSEP latency.

D.

Lower extremity SSEPs are not transmitted by the dorsal column of the spinal cord.

Correct answer is A

Taxonomy
I.C. BASIC: Organ-based Basic and Clinical Sciences, 1. Central and Peripheral Nervous Systems
II.A. ADVANCED: Basic Sciences, 1. Physics, Monitoring, and Anesthesia Delivery Devices

http://mycmecredit.com/ASA/ACEonline/printDiscussions.php

Page 1 of 108

Print ACE11A

5/25/14, 3:36 PM

QUESTION

2.

Cyclosporine toxicity is MOST commonly seen in which of the following solid organs?
A. Lung
B. Kidney
C. Pancreas
D. Heart

Comments on the incorrect answers


A.

Although cyclosporine has been reported to be associated with the development of bronchospasm,
cough, and dyspnea, pulmonary sequelae are not the most common complications.

C.

Toxicity of the pancreas does not occur with cyclosporine administration.

D.

Although cyclosporine has been reported to be associated with myocardial infarction, dysrhythmias,
and heart failure, cardiac complications are not the most common manifestation of cyclosporine
toxicity.

Correct answer is B

Taxonomy
I.B. BASIC: Clinical Sciences: Anesthesia Procedures, Methods, and Techniques, 1. Evaluation of the
Patient and Preoperative Preparation

http://mycmecredit.com/ASA/ACEonline/printDiscussions.php

Page 2 of 108

Print ACE11A

5/25/14, 3:36 PM

QUESTION

3.

Which of the following anesthetic agents undergoes the MOST metabolism?


A.

B.

C.

D.

Nitrous oxide

Isoflurane

Sevoflurane

Desflurane

Comments on the incorrect answers


A.

B.

D.

Nitrous oxide has not demonstrated any degree of metabolism when administered.

Only approximately 2% of the administered isoflurane is metabolized.

Only approximately 0.2% of the administered desflurane dose is metabolized.

Correct answer is C

Taxonomy
I.D. BASIC: Special Problems or Issues in Anesthesiology, 2. Ethics, Practice Management, and
Medicolegal Issues

http://mycmecredit.com/ASA/ACEonline/printDiscussions.php

Page 3 of 108

Print ACE11A

5/25/14, 3:36 PM

QUESTION

4.

In a preoperative anesthesia machine safety check, which of the following is MOST likely
affected by the presence of a check valve between the oxygen flush valve and the anesthetic
vaporizers?
A. Calibration of the oxygen analyzer
B. The low-pressure circuit leak test
C. The circle system flow test
D. The piped gas supply pressure check

Comments on the incorrect answers


A.

Calibration of the oxygen analyzer does not depend on the presence or absence of a check valve.

C.

The circle system flow test evaluates leaks in the circle system and the integrity of the unidirectional
valves and does not depend on the presence or absence of a check valve.

D.

The pressure check in the piped gas supply is part of the high-pressure system and is independent
of the presence of a check valve.

Correct answer is B

Taxonomy
I.B. BASIC: Clinical Sciences: Anesthesia Procedures, Methods, and Techniques, 7. Postoperative Period

http://mycmecredit.com/ASA/ACEonline/printDiscussions.php

Page 4 of 108

Print ACE11A

5/25/14, 3:36 PM

QUESTION

5.

A patient presents with pain associated with osteoarthritis. Treatment with which of the
following medications appears to be associated with the LOWEST risk of myocardial
infarction?
A.

B.

C.

D.

Diclofenac

Naproxen

Ibuprofen

Celecoxib

Comments on the incorrect answers


A.

C.

D.

Available evidence suggests that diclofenac is not associated with the lowest risk of myocardial
infarction.

Available evidence suggests that ibuprofen is not associated with the lowest risk of myocardial
infarction.

Celecoxib appears to be the safest of the COX-2 inhibitors but is not associated with the lowest risk
of myocardial infarction.

Correct answer is B

Taxonomy
II.D. ADVANCED: Clinical Subspecialties, 1. Painful Disease States

http://mycmecredit.com/ASA/ACEonline/printDiscussions.php

Page 5 of 108

Print ACE11A

5/25/14, 3:36 PM

QUESTION

6.

Administration of which of the following is LEAST likely to be appropriate in the setting of


acute intraoperative bronchospasm?
A. Isoflurane
B. Albuterol
C. Montelukast
D. Hydrocortisone

Comments on the incorrect answers


A.

Administration of isoflurane will improve bronchospasm by both deepening the level of anesthesia
and via a direct bronchodilating effect.

B.

The 2-agonist albuterol has direct bronchodilating properties and is a mainstay of therapy for acute
bronchospasm.

D.

Administration of steroids will decrease airway inflammation. The clinical effects are not immediate
but steroids will decrease the chance of recurrent or persistent bronchospasm and are appropriate
for administration in the acute phase.

Correct answer is C

Taxonomy
II.C. ADVANCED: Organ-based Basic and Clinical Sciences, 2. Respiratory System

http://mycmecredit.com/ASA/ACEonline/printDiscussions.php

Page 6 of 108

Print ACE11A

5/25/14, 3:36 PM

QUESTION

7.

Your patient has a preoperative cardiac arrest while moving himself from the stretcher to the
operating room table. According to the 2010 American Heart Association guidelines, the first
clinical response is to
A. intubate the trachea.
B. begin chest compressions.
C. deliver a precordial thump.
D. attach a monitor/defibrillator.

Comments on the incorrect answers


A.

Ventilation of the patients lungsin particular, placement of an advanced airway deviceshould not
delay or cause long interruptions in resuscitation.

C.

A precordial thump may be considered in patients with witnessed, monitored, unstable ventricular
tachycardia (VT), including pulseless VT, if a defibrillator is not immediately available.

D.

Attaching a monitor/defibrillator to the patient should be done as expeditiously as possible but should
not delay the start of resuscitation.

Correct answer is B

Taxonomy
II.C. ADVANCED: Organ-based Basic and Clinical Sciences, 3. Cardiovascular System

http://mycmecredit.com/ASA/ACEonline/printDiscussions.php

Page 7 of 108

Print ACE11A

5/25/14, 3:36 PM

QUESTION

8.

Which of the following is MOST likely associated with an increase in minimum alveolar
concentration (MAC) in humans?
A. Advanced age
B. Concurrent administration of intravenous local anesthetics
C. Preoperative administration of clonidine
D. Chronic alcohol abuse

Comments on the incorrect answers


A.

Advanced age is associated with a decrease in MAC.

B.

Concurrent administration of intravenous local anesthetics is associated with a decrease in MAC.

C.

Preoperative administration of 2 agonists such as clonidine is associated with a decrease in MAC.

Correct answer is D

Taxonomy
I.A. BASIC: Basic Sciences, 4. Pharmacology
II.A. ADVANCED: Basic Sciences, 2. Pharmacology

http://mycmecredit.com/ASA/ACEonline/printDiscussions.php

Page 8 of 108

Print ACE11A

5/25/14, 3:36 PM

QUESTION

9.

The BEST choice for a postoperative antiemetic medication for a patient with Parkinson
disease who has already been treated with intraoperative ondansetron and steroid is
A. droperidol.
B. ondansetron.
C. diphenhydramine.
D. prochlorperazine (Compazine).

Comments on the incorrect answers


A.

Droperidol, a butyrophenone, may aggravate Parkinson symptoms.

B.

Repeat dosing of ondansetron in a multimodal regimen for postoperative nausea and vomiting has
not been shown to be effective.

D.

Prochlorperazine, a phenothiazine, may aggravate Parkinson symptoms.

Correct answer is C

Taxonomy
I.B. BASIC: Clinical Sciences: Anesthesia Procedures, Methods, and Techniques, 7. Postoperative Period
II.C. ADVANCED: Organ-based Basic and Clinical Sciences, 1. Central and Peripheral Nervous Systems

http://mycmecredit.com/ASA/ACEonline/printDiscussions.php

Page 9 of 108

Print ACE11A

5/25/14, 3:36 PM

QUESTION

10.

Which of the following diseases is MOST closely associated with life-threatening


succinylcholine-induced hyperkalemia?
A. Myotonic dystrophy
B. Familial hypokalemic periodic paralysis
C. End-stage renal disease
D. Glycogen storage disease type 2 (acid maltase deficiency)

Comments on the incorrect answers


A.

Succinylcholine administration to patients with myotonic dystrophy might cause diffuse and
prolonged muscle contractions but not life-threatening hyperkalemia.

B.

Patients with familial hypokalemic periodic paralysis do not develop life-threatening hyperkalemia
with succinylcholine administration.

C.

Patients with end-stage renal disease do not exhibit a life-threatening hyperkalemic response to
succinylcholine.

Correct answer is D

Taxonomy
I.A. BASIC: Basic Sciences, 4. Pharmacology
II.A. ADVANCED: Basic Sciences, 2. Pharmacology

http://mycmecredit.com/ASA/ACEonline/printDiscussions.php

Page 10 of 108

Print ACE11A

5/25/14, 3:36 PM

QUESTION

11.

A patient undergoes placement of an axillary brachial plexus block using 1.25% mepivacaine.
Epinephrine at 5 g/mL is added to the local anesthetic solution to test for intravascular
injection. Which statement about the magnitude of the heart rate response to epinephrine is
MOST likely true?
A. It is independent of age.
B. It is blunted by pre-procedure administration of -blockade.
C. It is similar whether the block is performed before or after induction of general anesthesia.
D. When administered with mepivacaine, epinephrine does not affect heart rate.

Comments on the incorrect answers


A.

The magnitude of the heart rate increase is reduced as patients age, particularly after the age of 40.

C.

The heart rate and blood pressure responses to epinephrine are reduced when a patient undergoes
general anesthesia.

D.

The effect of epinephrine on heart rate when used as a marker for intravascular injection is
independent of the local anesthetic used.

Correct answer is B

Taxonomy
I.A. BASIC: Basic Sciences, 4. Pharmacology
II.B. ADVANCED: Clinical Sciences (Procedures, Methods, Techniques), 1. Regional Anesthesia

http://mycmecredit.com/ASA/ACEonline/printDiscussions.php

Page 11 of 108

Print ACE11A

5/25/14, 3:36 PM

QUESTION

12.

You are taking care of a patient with an acute abdomen who is entering the operating room
within the first few hours of septic shock. Which of the following is MOST likely
recommended at this point?
A. Measurement of serum lactate
B. Administration of bicarbonate to keep arterial pH above 7.3
C. Fluid restriction
D. Administration of phenylephrine for systemic hypotension

Comments on the incorrect answers


B.

Bicarbonate administration is not recommended unless arterial pH is below 7.15.

C.

By definition, a patient in septic shock is hypotensive. Delivery of an initial minimum of 30 mL/kg


crystalloid or an equivalent is recommended in the treatment of patients who are hypotensive or
have a serum lactate in excess of 4 mmol/L.

D.

Although evidence is not strong, administration of norepinephrine is recommended as the first-line


choice for vasopressor therapy, with addition of vasopressin if needed.

Correct answer is A

Taxonomy
II.D. ADVANCED: Clinical Subspecialties, 12. Critical Care

http://mycmecredit.com/ASA/ACEonline/printDiscussions.php

Page 12 of 108

Print ACE11A

5/25/14, 3:36 PM

QUESTION

13.

Which of the following is MOST likely a characteristic of postpolio syndrome?


A. Recurrent motor weakness
B. Persistent vomiting
C. Fever
D. Increased tolerance to opioids

Comments on the incorrect answers


B.

Dysphagia, not persistent vomiting, is associated with postpolio syndrome.

C.

Patients with postpolio syndrome are often cold intolerant. Fever is not associated with the
syndrome.

D.

Postpolio syndrome is associated with increased sensitivity to anesthetic agents, especially


sedatives.

Correct answer is A

Taxonomy
II.C. ADVANCED: Organ-based Basic and Clinical Sciences, 8. Neuromuscular Diseases and Disorders:
Clinical Science

http://mycmecredit.com/ASA/ACEonline/printDiscussions.php

Page 13 of 108

Print ACE11A

5/25/14, 3:36 PM

QUESTION

14.

A patient presents with visceral pelvic pain secondary to radiation therapy for rectal cancer.
Which of the following interventions is MOST likely to relieve this patients pain?
A. Celiac plexus block
B. Lumbar sympathetic block
C. Bilateral pudendal nerve blocks
D. Superior hypogastric plexus block

Comments on the incorrect answers


A.

A celiac plexus block will relieve visceral pain secondary to cancer of the upper abdomen but is
unlikely to relieve this patients visceral pelvic pain.

B.

A lumbar sympathetic block will relieve pain due to conditions such as lower extremity complex
regional pain syndrome and arterial insufficiency of the lower extremities but is unlikely to relieve
visceral pelvic pain.

C.

Bilateral pudendal nerve blocks are unlikely to relieve this patients visceral pain.

Correct answer is D

Taxonomy
II.D. ADVANCED: Clinical Subspecialties, 1. Painful Disease States

http://mycmecredit.com/ASA/ACEonline/printDiscussions.php

Page 14 of 108

Print ACE11A

5/25/14, 3:36 PM

QUESTION

15.

Which of the following conditions is MOST likely associated with an increase in carbon
monoxide diffusion capacity of the lung (DLCO)?
A.

B.

C.

D.

Chronic obstructive pulmonary disease (COPD)

Asthma

Sarcoidosis

Idiopathic pulmonary hypertension

Comments on the incorrect answers


A.

COPD is associated with a low DLCO.

C.

Sarcoidosis is associated with a low DLCO.

D.

Idiopathic pulmonary hypertension is associated with a low DLCO.

Correct answer is B

Taxonomy
I.C. BASIC: Organ-based Basic and Clinical Sciences, 2. Respiratory System
II.C. ADVANCED: Organ-based Basic and Clinical Sciences, 2. Respiratory System

http://mycmecredit.com/ASA/ACEonline/printDiscussions.php

Page 15 of 108

Print ACE11A

5/25/14, 3:36 PM

QUESTION

16.

Which of the following respiratory function parameters is MOST likely decreased in pregnant
patients during the third trimester?
A. Tidal volume
B. Vital capacity
C. Expiratory reserve volume
D. Forced expiratory volume in 1 second

Comments on the incorrect answers


A.

Tidal volume increases significantly during pregnancy.

B.

Vital capacity remains unchanged during pregnancy.

D.

Forced expiratory volume in 1 second remains unchanged during pregnancy.

Correct answer is C

Taxonomy
II.D. ADVANCED: Clinical Subspecialties, 3. Obstetric Anesthesia

http://mycmecredit.com/ASA/ACEonline/printDiscussions.php

Page 16 of 108

Print ACE11A

5/25/14, 3:36 PM

QUESTION

17.

A 36-year-old man who was admitted to the intensive care unit develops polyuria. His
laboratory values are significant for serum sodium 153 mmol/L, serum potassium 4.0 mmol/L,
serum osmolality 340 mOsm/kg, and urine osmolality 230 mOsm/kg. Which of the following is
the MOST likely reason for this increase in urine output?
A. Syndrome of inappropriate antidiuretic hormone secretion (SIADH)
B. Central diabetes insipidus
C. Cerebral salt wasting
D. Diabetic ketoacidosis

Comments on the incorrect answers


A.

SIADH is associated with hyponatremia and oliguria.

C.

Cerebral salt wasting is associated with hyponatremia.

D.

Serum sodium is normal or low in diabetic ketoacidosis.

Correct answer is B

Taxonomy
I.C. BASIC: Organ-based Basic and Clinical Sciences, 5. Renal and Urinary Systems/ Electrolyte Balance
II.C. ADVANCED: Organ-based Basic and Clinical Sciences, 7. Endocrine and Metabolic Systems

http://mycmecredit.com/ASA/ACEonline/printDiscussions.php

Page 17 of 108

Print ACE11A

5/25/14, 3:36 PM

QUESTION

18.

Which statement BEST describes neonatal injuries associated with forceps delivery?
A. Injuries are more common after outlet forceps than after midforceps delivery.
B. Facial nerve palsy secondary to forceps use is usually permanent.
C. There is an increased risk of brachial plexus injuries with forceps delivery.
D. Maternal epidural labor analgesia can significantly reduce these injuries.

Comments on the incorrect answers


A.

Neonatal injuries are more common after midforceps delivery.

B.

Facial nerve palsy secondary to forceps use usually resolves spontaneously.

D.

Maternal epidural labor analgesia has not been demonstrated to have any impact on the risk of
neonatal injuries with forceps deliveries.

Correct answer is C

Taxonomy
II.D. ADVANCED: Clinical Subspecialties, 3. Obstetric Anesthesia

http://mycmecredit.com/ASA/ACEonline/printDiscussions.php

Page 18 of 108

Print ACE11A

5/25/14, 3:36 PM

QUESTION

19.

A patient receiving general anesthesia develops severe hypotension, tachycardia, and


increased peak airway pressures approximately 10 minutes after placement of a left internal
jugular central venous catheter. Physical examination reveals decreased breath sounds on
the left. Which of the following is the MOST appropriate initial management?
A.

B.

C.

D.

Order a stat chest radiograph.

Withdraw the tracheal tube 3 cm.

Administer albuterol.

Perform needle decompression to the left chest.

Comments on the incorrect answers


A.

B.

C.

Ordering a stat chest radiograph would not be the most appropriate initial management for a patient
with severe hypotension, tachycardia, and increased peak airway pressures.

Withdrawing the tracheal tube would be appropriate management for a mainstem intubation.
Although intubation of the right mainstem bronchus would result in increased peak airway pressures
and diminished breath sounds on the left, it is unlikely to cause severe hypotension and tachycardia.

The administration of albuterol would be appropriate management for a patient with bronchospasm.
Although bronchospasm would cause increased peak airway pressure and could be associated with
tachycardia, it is unlikely to cause a unilateral decrease in breath sounds

Correct answer is D

Taxonomy
I.B. BASIC: Clinical Sciences: Anesthesia Procedures, Methods, and Techniques, 6. Complications
(Etiology, Prevention, Treatment)
II.C. ADVANCED: Organ-based Basic and Clinical Sciences, 2. Respiratory System

http://mycmecredit.com/ASA/ACEonline/printDiscussions.php

Page 19 of 108

Print ACE11A

5/25/14, 3:36 PM

QUESTION

20.

Which statement about the clinical presentation of heparin-induced thrombocytopenia (HIT)


is MOST likely true?
A. Thrombosis occurs in less than 25% of patients with HIT.
B. Pulmonary embolism is rare in patients with HIT.
C. Significant thrombosis occurs only if the platelet count is less than 100,000/L.
D. Spontaneous bleeding is rare in patients with HIT.

Comments on the incorrect answers


A.

Thrombosis occurs in up to 75% of patients with HIT.

B.

The most common presentation of HIT is pulmonary embolism.

C.

Significant thrombosis can occur with platelet counts in excess of 100,000/L and may even precede
the development of thrombocytopenia.

Correct answer is D

Taxonomy
I.C. BASIC: Organ-based Basic and Clinical Sciences, 6. Hematologic System
II.C. ADVANCED: Organ-based Basic and Clinical Sciences, 6. Hematologic System

http://mycmecredit.com/ASA/ACEonline/printDiscussions.php

Page 20 of 108

Print ACE11A

5/25/14, 3:36 PM

QUESTION

21.

Which of the following radiological signs is MOST likely to be present in a child with
laryngotracheobronchitis?
A. Thumb sign
B. Anteater nose sign
C. Hampton hump
D. Steeple sign

Comments on the incorrect answers


A.

The thumb (or thumbprint) sign is seen on lateral neck radiographs in children with epiglottitis.

B.

The anteater nose sign is seen on radiographs of the foot/ankle in calcaneonavicular tarsal coalition
(the abnormal fusion of 2 or more tarsal bones).

C.

Hampton hump is a wedge-shaped area of pulmonary consolidation that is sometimes seen on chest
radiographs in the presence of a pulmonary embolus.

Correct answer is D

Taxonomy
II.D. ADVANCED: Clinical Subspecialties, 2. Pediatric Anesthesia

http://mycmecredit.com/ASA/ACEonline/printDiscussions.php

Page 21 of 108

Print ACE11A

5/25/14, 3:36 PM

QUESTION

22.

Which of the following parameters is MOST likely to be increased after release of a lower
extremity tourniquet during total knee arthroplasty?
A. Central venous pressure
B. Systemic vascular resistance
C. Serum lactate level
D. Core body temperature

Comments on the incorrect answers


A.

Central venous pressure would be expected to decrease after release of a lower extremity tourniquet
during total knee arthroplasty.

B.

Tourniquet release is associated with a decrease in systemic vascular resistance.

D.

Core body temperature would be expected to decrease after release of a lower extremity tourniquet
during total knee arthroplasty.

Correct answer is C

Taxonomy
II.D. ADVANCED: Clinical Subspecialties, 8. Orthopedic Anesthesia; Tourniquet Management,
Complications, Regional vs. General Anesthesia

http://mycmecredit.com/ASA/ACEonline/printDiscussions.php

Page 22 of 108

Print ACE11A

5/25/14, 3:36 PM

QUESTION

23.

Which of the following sets of arterial blood gases is MOST typical of a patient with severe
chronic obstructive pulmonary disease (COPD) seen in the preoperative evaluation clinic?

PaO2 (mm Hg)

PaCO2 (mm Hg)

pH

HCO3- (mEq/L)

(A)

85

29

7.40

19

(B)

64

60

7.35

31

(C)

100

28

7.52

24

(D)

100

40

7.40

24

A.

B.

(A)

(B)

http://mycmecredit.com/ASA/ACEonline/printDiscussions.php

Page 23 of 108

Print ACE11A

5/25/14, 3:36 PM

C.

D.

(C)

(D)

Comments on the incorrect answers


A.

C.

D.

These results show a normal pH, a metabolic acidosis, and a respiratory alkalosis. They are not
typical of a patient with COPD.

These results show a primary respiratory alkalosis with alkalemia. This situation is unlikely to be
seen in a patient with COPD.

These are normal results.

Correct answer is B

Taxonomy
II.C. ADVANCED: Organ-based Basic and Clinical Sciences, 2. Respiratory System

http://mycmecredit.com/ASA/ACEonline/printDiscussions.php

Page 24 of 108

Print ACE11A

5/25/14, 3:36 PM

QUESTION

24.

Which of the following management strategies is MOST likely to improve perioperative


outcome in patients with chronic obstructive pulmonary disease (COPD)?
A. Preoperative cardiopulmonary conditioning
B. Use of general (versus regional) anesthesia
C. Application of high levels of positive end-expiratory pressure (PEEP) if mechanical ventilation is
necessary
D. Use of high tidal volumes during mechanical ventilation

Comments on the incorrect answers


B.

In patients with COPD, avoidance of general anesthesia and mechanical ventilation is preferable.
Regional anesthesia is a preferred option if the procedure is suitable for a regional technique.

C.

Structural changes in the lungs of patients with COPD mean that they are at increased risk for
ventilator-induced lung injury. Mechanical ventilation should aim to avoid pulmonary hyperinflation
and the development of intrinsic PEEP.

D.

Mechanical ventilation of patients with COPD should aim to avoid pulmonary hyperinflation and the
development of intrinsic PEEP.

Correct answer is A

Taxonomy

http://mycmecredit.com/ASA/ACEonline/printDiscussions.php

Page 25 of 108

Print ACE11A

5/25/14, 3:36 PM

II.C. ADVANCED: Organ-based Basic and Clinical Sciences, 2. Respiratory System

http://mycmecredit.com/ASA/ACEonline/printDiscussions.php

Page 26 of 108

Print ACE11A

5/25/14, 3:36 PM

QUESTION

25.

The following clinical scenarios describe critically ill patients requiring mechanical
ventilation who have loss of corneal reflexes but intact cardiac function. In which case would
a formal assessment for determination of brain death MOST likely be carried out?
A. A 45-year-old man with traumatic brain injury after a motor vehicle crash whose core temperature
is 31.8C
B. A 60-year-old woman with coma of undetermined etiology
C. A 58-year old man with subarachnoid hemorrhage who received a single dose of atracurium for
intubation 4 days earlier
D. A 69-year-old woman after in-hospital cardiac arrest who has decerebrate posturing

Comments on the incorrect answers


A.

A patient should have a core temperature of at least 36.5C degrees prior to brain death
assessment.

B.

The cause of coma should be known before assessment for the presence of brain death is
performed.

D.

Decerebrate posturing is not seen in patients with brain death as it implies the presence of some
brain stem function.

Correct answer is C

Taxonomy
II.C. ADVANCED: Organ-based Basic and Clinical Sciences, 1. Central and Peripheral Nervous Systems
II.E. ADVANCED: Special Problems or Issues in Anesthesiology, 2. Organ Donors: Pathophysiology and
Clinical Management

http://mycmecredit.com/ASA/ACEonline/printDiscussions.php

Page 27 of 108

Print ACE11A

5/25/14, 3:36 PM

QUESTION

26.

Which statement about carbon monoxide (CO) formation as a result of administration of


inhaled anesthetic agents is MOST likely true?
A. Increased temperature in the canister of carbon dioxide (CO2) absorbent is predictive of CO
production.
B. Exposure to desiccated CO2 absorbent is a necessary step in producing CO.
C. Desflurane produces the lowest amounts of CO.
D. The more acidic the CO2 absorbent, the greater the amount of CO produced.

Comments on the incorrect answers


A.

CO2 absorbent temperature is a poor predictor of CO formation.

C.

Desflurane produces the highest amounts of CO.

D.

CO2 absorbents that are highly alkaline are capable of producing the highest amounts of CO in the
presence of potent inhaled anesthetic agents.

Correct answer is B

Taxonomy
I.A. BASIC: Basic Sciences, 2. Physics, Monitoring, and Anesthesia Delivery Devices

http://mycmecredit.com/ASA/ACEonline/printDiscussions.php

Page 28 of 108

Print ACE11A

5/25/14, 3:36 PM

QUESTION

27.

Which statement about the management of a patient undergoing parathyroidectomy for a


single parathyroid adenoma is MOST likely true?
A. The nadir of serum calcium occurs within 30 minutes of parathyroid adenoma resection.
B. Hungry bone syndrome occurs when bone rapidly remineralizes.
C. Total intravenous anesthesia with propofol can interfere with the rapid parathormone assay.
D. Injury of one recurrent laryngeal nerve will result in immediate airway obstruction upon
extubation.

Comments on the incorrect answers


A.

The nadir of serum calcium occurs 3 to 10 days after successful parathyroid adenoma resection.

C.

Total intravenous anesthesia has not been shown to interfere with rapid parathyroid hormone assay.

D.

Injury of a single recurrent laryngeal nerve will result in hoarseness.

Correct answer is B

Taxonomy
I.C. BASIC: Organ-based Basic and Clinical Sciences, 7. Endocrine and Metabolic Systems
II.C. ADVANCED: Organ-based Basic and Clinical Sciences, 7. Endocrine and Metabolic Systems

http://mycmecredit.com/ASA/ACEonline/printDiscussions.php

Page 29 of 108

Print ACE11A

5/25/14, 3:36 PM

QUESTION

28.

Which statement about local anesthetic toxicity is MOST likely true?


A. Respiratory acidosis decreases the risk of central nervous system toxicity.
B. Vasoconstriction is the predominant vascular response.
C. Prior administration of a benzodiazepine decreases the likelihood of a seizure.
D.The ratio of the dosage that induces cardiovascular collapse to the dosage that will produce
seizures is higher for bupivacaine than other local anesthetics.

Comments on the incorrect answers


A.

Acidosis, whether respiratory or metabolic, increases the risk of local anestheticinduced central
nervous system toxicity.

B.

Although vasoconstriction may be present at low concentrations of local anesthetics, vasodilation will
be the predominant vascular response in circumstances of local anesthetic toxicity.

D.

The ratio of the dosage that induces cardiovascular collapse to the dosage that will produce seizures
is lower for bupivacaine than other local anesthetics.

Correct answer is C

Taxonomy
I.A. BASIC: Basic Sciences, 4. Pharmacology

http://mycmecredit.com/ASA/ACEonline/printDiscussions.php

Page 30 of 108

Print ACE11A

5/25/14, 3:36 PM

QUESTION

29.

Which of the following ipsilateral signs is the BEST indicator of complete sympathetic
blockade of the upper extremity following a stellate ganglion block?
A. Exophthalmos
B. Miosis
C. Nasal congestion
D. Increased forearm skin temperature

Comments on the incorrect answers


A.

Stellate ganglion block produces ipsilateral enophthalmos, not exophthalmos.

B.

Although successful stellate ganglion block produces ipsilateral miosis, this does not guarantee
sympathetic blockade of the upper extremity.

C.

Although successful stellate ganglion block produces ipsilateral nasal congestion, this does not
guarantee sympathetic blockade of the upper extremity.

Correct answer is D

Taxonomy
II.B. ADVANCED: Clinical Sciences (Procedures, Methods, Techniques), 1. Regional Anesthesia
II.C. ADVANCED: Organ-based Basic and Clinical Sciences, 1. Central and Peripheral Nervous Systems

http://mycmecredit.com/ASA/ACEonline/printDiscussions.php

Page 31 of 108

Print ACE11A

5/25/14, 3:36 PM

QUESTION

30.

A 42-year-old woman presents to the emergency department with acute back pain after
having been lost to follow-up for addiction treatment 2 months ago. She has a past medical
history of intravenous heroin addiction in remission and is currently being managed with
buprenorphine/naloxone therapy. The patient appears calm, drowsy, and comfortable. She is
asking for her usual dose of buprenorphine as she has not yet taken her medication today.
Which of the following is MOST indicative of intravenous heroin relapse if this patient is
given buprenorphine?
A. Respiratory depression
B. Sedation
C. Mydriasis
D. Headache

Comments on the incorrect answers


A.

Respiratory depression is a symptom of opioid overdose. Introduction of buprenorphine in the setting


of intravenous heroin use precipitates opioid withdrawal.

B.

Sedation is a symptom of opioid overdose. Introduction of buprenorphine in the setting of


intravenous heroin use precipitates opioid withdrawal.

D.

Headache is a potential side effect of buprenorphine but does not indicate relapse to intravenous
heroin use.

Correct answer is C

Taxonomy
II.A. ADVANCED: Basic Sciences, 2. Pharmacology

http://mycmecredit.com/ASA/ACEonline/printDiscussions.php

Page 32 of 108

Print ACE11A

5/25/14, 3:36 PM

QUESTION

31.

A decrease in the maximum dose of lidocaine for liposuction is MOST appropriate for
patients taking which of the following medications?
A. Nicardipine
B. Carbamazepine
C. St Johns wort
D. Omeprazole

Comments on the incorrect answers


B.

Carbamazepine induces cytochrome P450 3A4, which metabolizes lidocaine. Greater activity of the
enzyme will result in more rapid elimination of lidocaine.

C.

St Johns wort induces cytochrome P450 3A4, which metabolizes lidocaine. Greater activity of the
enzyme will result in more rapid elimination of lidocaine.

D.

Omeprazole induces cytochrome P450 1A2, which metabolizes lidocaine. Greater activity of the
enzyme will result in more rapid elimination of lidocaine.

Correct answer is A

Taxonomy
II.D. ADVANCED: Clinical Subspecialties, 5. Anesthesia for Plastic Surgery, Liposuction

http://mycmecredit.com/ASA/ACEonline/printDiscussions.php

Page 33 of 108

Print ACE11A

5/25/14, 3:36 PM

QUESTION

32.

Assuming an inadvertent dural puncture occurs during placement of an epidural catheter,


which of the following conditions will MOST likely be associated with a higher risk of
postdural puncture headache (PDPH)?
A.

B.

C.

D.

Morbid obesity

Patient age greater than 60 years

Use of saline to identify the epidural space

Previous PDPH

Comments on the incorrect answers


A.

B.

C.

Morbid obesity is associated with a decreased risk of PDPH.

The risk of PDPH is lower in patients 60 years old and older.

Use of saline to locate the epidural space is not reported to be a risk factor for development of
PDPH.

Correct answer is D

http://mycmecredit.com/ASA/ACEonline/printDiscussions.php

Page 34 of 108

Print ACE11A

5/25/14, 3:36 PM

QUESTION

33.

A 56-year-old man with superior vena cava syndrome presents for a cervical lymph node
biopsy under general anesthesia. Which of the following is MOST likely an important
component of the management strategy?
A. Armored tracheal tube
B. Large-bore intravenous (IV) access in forearm
C. Preoperative diuresis
D. Trendelenburg position

Comments on the incorrect answers


B.

Upper extremity IV access should be avoided in patients with superior vena cava syndrome due to
interrupted blood flow from the upper body.

C.

Maintenance of preload is recommended for patients with superior vena cava syndrome.

D.

Trendelenburg position increases upper extremity and cerebral venous engorgement and should be
avoided.

Correct answer is A

Taxonomy
II.C. ADVANCED: Organ-based Basic and Clinical Sciences, 3. Cardiovascular System

http://mycmecredit.com/ASA/ACEonline/printDiscussions.php

Page 35 of 108

Print ACE11A

5/25/14, 3:36 PM

QUESTION

34.

Abrupt withdrawal of levodopa places the patient with Parkinson disease at increased risk for
each of the following EXCEPT
A. chest wall rigidity.
B. aspiration.
C. autonomic instability.
D. nausea.

Comments on the incorrect answers


A.

Chest wall rigidity is a symptom of Parkinson disease and can occur after acute withdrawal of
levodopa.

B.

Acute withdrawal of levodopa is associated with exacerbation of Parkinson symptoms, one of which
is dysphagia, leading to increased risk of aspiration.

C.

Acute discontinuation of levodopa has been reported to precipitate autonomic instability, altered
mental status, muscle rigidity, and fever.

Correct answer is D

Taxonomy
II.C. ADVANCED: Organ-based Basic and Clinical Sciences, 1. Central and Peripheral Nervous Systems

http://mycmecredit.com/ASA/ACEonline/printDiscussions.php

Page 36 of 108

Print ACE11A

5/25/14, 3:36 PM

QUESTION

35.

Which of the following is LEAST likely to decrease allogenic blood transfusion during
elective hip replacement surgery?
A. Hypotensive epidural technique
B. Selective mild hypothermia (35C)
C. Preoperative erythropoietin therapy
D. Normovolemic hemodilution

Comments on the incorrect answers


A.

Hypotensive epidural technique reduces blood loss.

C.

Preoperative erythropoietin therapy reduces the need for blood transfusion.

D.

Normovolemic hemodilution reduces the need for blood transfusion.

Correct answer is B

Taxonomy
I.C. BASIC: Organ-based Basic and Clinical Sciences, 6. Hematologic System
II.C. ADVANCED: Organ-based Basic and Clinical Sciences, 6. Hematologic System

http://mycmecredit.com/ASA/ACEonline/printDiscussions.php

Page 37 of 108

Print ACE11A

5/25/14, 3:36 PM

QUESTION

36.

Which of the following statements about transfusion-associated graft versus host disease
(TA-GVHD) is MOST likely true?
A. The risk is increased with administration of directed donor blood products.
B. It most commonly presents within the first 48 hours following transfusion.
C. Patients remain afebrile.
D. It is more common in children.

Comments on the incorrect answers


B.

TA-GVHD generally presents 1 to 6 weeks after transfusion.

C.

Fever is one of the presenting manifestations of TA-GVHD.

D.

TA-GVHD occurs most commonly in patients over 60 years of age.

Correct answer is A

Taxonomy
I.C. BASIC: Organ-based Basic and Clinical Sciences, 6. Hematologic System
II.C. ADVANCED: Organ-based Basic and Clinical Sciences, 6. Hematologic System

http://mycmecredit.com/ASA/ACEonline/printDiscussions.php

Page 38 of 108

Print ACE11A

5/25/14, 3:36 PM

QUESTION

37.

Which of the following factors has the STRONGEST association with the development of a
common peroneal neuropathy following surgery in the lithotomy position?
A. Nonsmoking patients
B. Low body mass index
C. Procedures of short duration
D. Spondylolisthesis at L1-2

Comments on the incorrect answers


A.

In a large retrospective review, patients with this neuropathy were more likely to be smokers.

C.

In a large retrospective review, patients with a common peroneal neuropathy were more likely to
have undergone longer procedures.

D.

The common peroneal nerve contains nerve fibers originating at L4 to S2, and a motor neuropathy of
this nerve can result in an inability to dorsiflex the foot (foot drop).

Correct answer is B

Taxonomy
I.B. BASIC: Clinical Sciences: Anesthesia Procedures, Methods, and Techniques, 6. Complications
(Etiology, Prevention, Treatment)

http://mycmecredit.com/ASA/ACEonline/printDiscussions.php

Page 39 of 108

Print ACE11A

5/25/14, 3:36 PM

QUESTION

38.

Which of the following is BEST avoided in the perioperative period in a patient with multiple
sclerosis (MS)?
A. Hyperthermia
B. Nondepolarizing neuromuscular blockers
C. Corticosteroids
D. Epidural anesthesia

Comments on the incorrect answers


B.

Even though there can be a variable response to nondepolarizing neuromuscular blockers in


patients with MS, their use is considered safe.

C.

Acute exacerbations of MS symptoms are treated with immunosuppressant medications, including


corticosteroids.

D.

Epidural anesthesia in patients with MS is generally considered to be associated with less risk than
spinal anesthesia; however, both have been used successfully in patients with MS.

Correct answer is A

Taxonomy
I.B. BASIC: Clinical Sciences: Anesthesia Procedures, Methods, and Techniques, 2. Regional Anesthesia
II.C. ADVANCED: Organ-based Basic and Clinical Sciences, 8. Neuromuscular Diseases and Disorders:
Clinical Science

http://mycmecredit.com/ASA/ACEonline/printDiscussions.php

Page 40 of 108

Print ACE11A

5/25/14, 3:36 PM

QUESTION

39.

Which of the following agents is likely to produce the LONGEST seizure duration when used
to induce anesthesia for electroconvulsive therapy (ECT)?
A.

B.

C.

D.

Methohexital

Etomidate

Propofol

Ketamine

Comments on the incorrect answers


A.

C.

D.

Methohexital exerts little to no effect on seizure duration in patients receiving ECT.

In patients receiving ECT, propofol has been shown to shorten seizure duration significantly unless
given in subhypnotic doses.

Ketamine is associated with a significantly shortened seizure duration in patients receiving ECT.

Correct answer is B

Taxonomy
II.E. ADVANCED: Special Problems or Issues in Anesthesiology, 1. Electroconvulsive Therapy

http://mycmecredit.com/ASA/ACEonline/printDiscussions.php

Page 41 of 108

Print ACE11A

5/25/14, 3:36 PM

QUESTION

40.

Which of the following is MOST likely to occur in a patient who develops a moderate sized
pulmonary embolism during surgery?
A. Increased dead space
B. Decreased peak inspiratory pressure
C. Bradycardia
D. Increased end-tidal carbon dioxide

Comments on the incorrect answers


B.

Bronchospasm, associated with the release of serotonin and thromboxane A2 occurring as a result
of platelet activation, commonly produces an increase in peak inspiratory pressure in patients with
pulmonary embolism.

C.

Tachycardia is reported to occur in 71% of patients with pulmonary embolism.

D.

Because end-tidal gas results from a mixture of gas from normal alveoli and from alveoli lacking
perfusion as a result of the embolus, end-tidal carbon dioxide concentrations decrease with
pulmonary embolism.

Correct answer is A

Taxonomy
II.C. ADVANCED: Organ-based Basic and Clinical Sciences, 3. Cardiovascular System

http://mycmecredit.com/ASA/ACEonline/printDiscussions.php

Page 42 of 108

Print ACE11A

5/25/14, 3:36 PM

QUESTION

41.

A 63-year-old man is scheduled for video-assisted left lower lobe lobectomy for
adenocarcinoma of the lung. He had a drug-eluting stent placed in his right coronary artery 5
months ago. He currently takes daily clopidogrel and aspirin. The MOST appropriate
perioperative management at this time is to
A. postpone the surgery for 1 month.
B. discontinue clopidogrel for 7 days prior to surgery.
C. discontinue both clopidogrel and aspirin for 7 days prior to surgery.
D. continue clopidogrel and aspirin throughout the perioperative period.

Comments on the incorrect answers


A.

Elective procedures should be postponed if possible for 12 months following placement of a drugeluting coronary stent. This case should proceed as the patient has lung cancer.

B.

Clopidogrel and aspirin should be continued throughout the perioperative period in a patient with a
drug-eluting stent in place for less than 12 months.

C.

Clopidogrel and aspirin should be continued throughout the perioperative period in a patient with a
drug-eluting stent in place for less than 12 months.

Correct answer is D

Taxonomy
I.B. BASIC: Clinical Sciences: Anesthesia Procedures, Methods, and Techniques, 1. Evaluation of the
Patient and Preoperative Preparation
II.C. ADVANCED: Organ-based Basic and Clinical Sciences, 3. Cardiovascular System

http://mycmecredit.com/ASA/ACEonline/printDiscussions.php

Page 43 of 108

Print ACE11A

5/25/14, 3:36 PM

QUESTION

42.

A 65-year-old woman with a 100 pack-year history of tobacco use presents for a
cholecystectomy. Which of the following would be LEAST consistent with cor pulmonale
secondary to chronic obstructive pulmonary disease?
A. Lower extremity edema
B. Elevated pulmonary artery occlusion pressure
C. Jugular venous distension
D. Pulmonary arterial hypertension

Comments on the incorrect answers


A.

Lower extremity edema is a sign of cor pulmonale.

C.

Jugular venous distension is a sign of cor pulmonale.

D.

Pulmonary arterial hypertension is a sign of cor pulmonale.

Correct answer is B

Taxonomy
II.C. ADVANCED: Organ-based Basic and Clinical Sciences, 2. Respiratory System
II.C. ADVANCED: Organ-based Basic and Clinical Sciences, 3. Cardiovascular System

http://mycmecredit.com/ASA/ACEonline/printDiscussions.php

Page 44 of 108

Print ACE11A

5/25/14, 3:36 PM

QUESTION

43.

Which of the following is MOST likely to indicate an increased risk in adult patients for
perioperative adrenal insufficiency?
A. A high normal plasma preoperative cortisol level
B. Prednisone therapy consisting of 5 mg every other day for 3 doses
C. Testosterone therapy for andropause in men
D. A septic patient induced with etomidate

Comments on the incorrect answers


A.

A high normal plasma cortisol level is not indicative of increased risk for adrenal insufficiency.

B.

Short-term, low-dose, every-other-day glucocorticoid treatment does not increase the risk of adrenal
insufficiency.

C.

Testosterone therapy for androgen deficiency does not increase the risk of adrenal insufficiency.

Correct answer is D

Taxonomy
I.B. BASIC: Clinical Sciences: Anesthesia Procedures, Methods, and Techniques, 1. Evaluation of the
Patient and Preoperative Preparation
II.C. ADVANCED: Organ-based Basic and Clinical Sciences, 7. Endocrine and Metabolic Systems

http://mycmecredit.com/ASA/ACEonline/printDiscussions.php

Page 45 of 108

Print ACE11A

5/25/14, 3:36 PM

QUESTION

44.

Which statement about monitoring patients having surgery performed in the sitting position
is MOST likely true?
A. The circle of Willis is level with the external auditory meatus.
B. Approximately 90% of patients have an intact circle of Willis.
C. Blood pressure measured in the lower extremity may be as much as 35 mm Hg higher than at the
external auditory meatus.
D. There is a 1.25 mm Hg decrease in blood pressure for each 1 cm height above the level at which
the blood pressure is being measured.

Comments on the incorrect answers


A.

In the sitting position, the base of the brain is commonly estimated to be level with the external
auditory meatus. In this position, the circle of Willis is more than 10 cm above the base of the brain
and the external auditory meatus.

B.

Autopsy studies show that more than 45% of the adult population has an incomplete circle of Willis.

D.

There is approximately a 0.77 mm Hg decrease in blood pressure for each cm in height above the
level at which the blood pressure is being measured.

Correct answer is C

Taxonomy
II.A. ADVANCED: Basic Sciences, 1. Physics, Monitoring, and Anesthesia Delivery Devices
II.C. ADVANCED: Organ-based Basic and Clinical Sciences, 1. Central and Peripheral Nervous Systems

http://mycmecredit.com/ASA/ACEonline/printDiscussions.php

Page 46 of 108

Print ACE11A

5/25/14, 3:36 PM

QUESTION

45.

Which statement about malignant hyperthermia (MH) is MOST likely true?


A. The risk of MH is lower in children than in adults.
B. The first sign of an MH episode is an increase in temperature.
C. The first dose of dantrolene is 5 mg/kg intravenously (IV).
D. Central core disease is clearly linked to MH.

Comments on the incorrect answers


A.

The rate of MH reported in children is 1:3,000 to 1:15,000 anesthetics. The reported rate in adults is
1 in 50,000 anesthetics.

B.

The first sign of MH is tachypnea (if breathing spontaneously) or tachycardia. Hyperthermia is


considered an intermediate sign.

C.

Dantrolene dosing begins at 2.5 mg/kg IV.

Correct answer is D

Taxonomy
II.A. ADVANCED: Basic Sciences, 2. Pharmacology

http://mycmecredit.com/ASA/ACEonline/printDiscussions.php

Page 47 of 108

Print ACE11A

5/25/14, 3:36 PM

QUESTION

46.

Which of the following abnormal movements is MOST likely to occur in children during
induction with propofol?
A. Opisthotonus (an arching position exemplified by severe hyperextension of the back and neck)
B. Choreoathetoid movements
C. Generalized tonicclonic activity
D. Myoclonus

Comments on the incorrect answers


A.

A few case reports describe opisthotonus after propofol administration.

B.

Case reports of choreoathetoid movements are rare.

C.

Tonicclonic seizure-like movements have been reported to occur in 5.5% of adult patients, rarely
in children. Electroencephalography-documented seizures are rare after propofol.

Correct answer is D

Taxonomy
I.A. BASIC: Basic Sciences, 4. Pharmacology
II.D. ADVANCED: Clinical Subspecialties, 2. Pediatric Anesthesia

http://mycmecredit.com/ASA/ACEonline/printDiscussions.php

Page 48 of 108

Print ACE11A

5/25/14, 3:36 PM

QUESTION

47.

A 78-year-old patient and his family report that he has had some difficulty concentrating and
remembering things. He is scheduled for prostate surgery and states he may refuse to have
the surgery if it means he wont be himself afterward. Which of the following statements
about this situation is MOST likely true?
A. Delirium occurs in less than 2% of patients who are over 65 years.
B. Administration of a sedative will help prevent delirium.
C. Preexisting mild dementia does not increase the risk of postoperative cognitive change.
D. Preoperative treatment of cognitive impairment may decrease the risk of delirium.

Comments on the incorrect answers


A.

Delirium is a serious postoperative complication occurring in 5% to 15% of elderly patients after


general anesthesia and in up to 64% of elderly patients after total joint replacement under spinal
anesthesia.

B.

Risk factors for postoperative delirium include the use of central nervous system depressant drugs,
particularly benzodiazepines.

C.

Patients with preexisting cognitive disorders have a higher risk for postoperative cognitive
complications than other patients. The most common complication is delirium, which occurs in about
40% of elderly surgical patients with preexisting cognitive impairment.

Correct answer is D

Taxonomy
II.D. ADVANCED: Clinical Subspecialties, 11. Geriatric Anesthesia/Aging

http://mycmecredit.com/ASA/ACEonline/printDiscussions.php

Page 49 of 108

Print ACE11A

5/25/14, 3:36 PM

QUESTION

48.

Which of the following medications is the MOST appropriate therapy for a patient with
extrapyramidal symptoms associated with the administration of metoclopramide?
A. Flumazenil
B. Neostigmine
C. Physostigmine
D. Diphenhydramine

Comments on the incorrect answers


A.

Flumazenil is a benzodiazepine antagonist and would not be expected to be effective in the


management of extrapyramidal symptoms.

B.

Neostigmine is an acetylcholinesterase inhibitor. Its administration results in an increase in


acetylcholine and would not be expected to be effective in the management of extrapyramidal
symptoms.

C.

Physostigmine is an acetylcholinesterase inhibitor. Its administration results in an increase in


acetylcholine and would not be expected to be effective in the management of extrapyramidal
symptoms.

Correct answer is D

Taxonomy
I.B. BASIC: Clinical Sciences: Anesthesia Procedures, Methods, and Techniques, 7. Postoperative Period

http://mycmecredit.com/ASA/ACEonline/printDiscussions.php

Page 50 of 108

Print ACE11A

5/25/14, 3:36 PM

QUESTION

49.

According to the American Heart Associations pediatric basic life support (BLS) guidelines,
which of the following is the MOST appropriate first step in management for an unresponsive
and apneic pediatric patient?
A. Open the airway
B. Start chest compressions
C. Check for a pulse
D. Start rescue breathing

Comments on the incorrect answers


A.

According to the BLS guidelines, opening the airway is not the first step in the management for an
unresponsive and apneic pediatric patient.

B.

According to the BLS guidelines, starting chest compressions is not the first step in the management
for an unresponsive and apneic pediatric patient.

D.

According to the BLS guidelines, starting rescue breathing is not the first step in the management for
an unresponsive and apneic pediatric patient.

Correct answer is C

Taxonomy
II.C. ADVANCED: Organ-based Basic and Clinical Sciences, 3. Cardiovascular System

http://mycmecredit.com/ASA/ACEonline/printDiscussions.php

Page 51 of 108

Print ACE11A

5/25/14, 3:36 PM

QUESTION

50.

A 62-year-old man with a 30 pack-year history of smoking presents with a chronic cough and
occasional blood-tinged sputum. A computed tomography scan of the thorax reveals
mediastinal lymphadenopathy and a right upper lobe lesion. The patient is scheduled for a
diagnostic mediastinoscopy with lymph node biopsy. Which of the following would be
considered the MOST appropriate site for an arterial line placement to quickly detect
compression caused by the mediastinoscope during this procedure?
A. Left lower extremity
B. Right upper extremity
C. Left upper extremity
D. Right lower extremity

Comments on the incorrect answers


A.

The left lower extremity would not be the most appropriate site for placement of an arterial line.

C.

The left upper extremity would not be the most appropriate site for placement of an arterial line.

D.

The right lower extremity would not be the most appropriate site for placement of an arterial line.

Correct answer is B

Taxonomy
II.A. ADVANCED: Basic Sciences, 1. Physics, Monitoring, and Anesthesia Delivery Devices
II.C. ADVANCED: Organ-based Basic and Clinical Sciences, 2. Respiratory System

http://mycmecredit.com/ASA/ACEonline/printDiscussions.php

Page 52 of 108

Print ACE11A

5/25/14, 3:36 PM

QUESTION

51.

A 67-year-old man with adenocarcinoma of the lung is scheduled for a thoracotomy and right
upper lobectomy. Which of the following preoperative values is MOST predictive of
postoperative pulmonary complications?
A.

B.

C.

D.

Maximal oxygen consumption (V O2max) = 10 mL kg1 min1

Diffusing capacity of the lung for carbon monoxide (DLCO) = 80% of predicted

PaCO2 = 40 mm Hg

FEV1 = 80% of predicted

Comments on the incorrect answers


B.

C.

D.

Predicted postoperative DLCO less than 40% is predictive of perioperative, but not long-term,
mortality. Resection of only the right upper lobe should not decrease DLCO to less than 40% of
predicted from a preoperative value of 80% of predicted.

A preoperative PaCO2 level greater than 60 mm Hg has been a traditional cutoff for pulmonary
resection. However, alone, it is not a sensitive predictor of perioperative complications.

An FEV1 less than 40% is predictive of perioperative respiratory complications.

Correct answer is A

Taxonomy
II.C. ADVANCED: Organ-based Basic and Clinical Sciences, 2. Respiratory System

http://mycmecredit.com/ASA/ACEonline/printDiscussions.php

Page 53 of 108

Print ACE11A

5/25/14, 3:36 PM

QUESTION

52.

Which of the following drugs is MOST likely safe in a patient with acute intermittent
porphyria?
A. Etomidate
B. Succinylcholine
C. Ketorolac
D. Methohexital

Comments on the incorrect answers


A.

Etomidate is listed as being unsafe in patients with acute intermittent porphyria.

C.

Ketorolac is listed as being possibly porphyrogenic.

D.

Methohexital is listed as being unsafe in patients with acute intermittent porphyria.

Correct answer is B

Taxonomy
II.C. ADVANCED: Organ-based Basic and Clinical Sciences, 6. Hematologic System

http://mycmecredit.com/ASA/ACEonline/printDiscussions.php

Page 54 of 108

Print ACE11A

5/25/14, 3:36 PM

QUESTION

53.

You are called to the magnetic resonance imaging (MRI) suite to assist with managing a
patient who has experienced a cardiac arrest that has occurred inside the scanner. Which of
the following pieces of equipment is MOST likely safe to bring into the MRI room?
A. Defibrillator
B. Video laryngoscope
C. Laryngeal mask airway
D. Standard code cart

Comments on the incorrect answers


A.

The use of a defibrillator is not safe inside the MRI environment.

B.

The use of a video laryngoscope is not safe inside the MRI environment.

D.

The use of a code cart is not safe inside the MRI environment.

Correct answer is C

Taxonomy
II.E. ADVANCED: Special Problems or Issues in Anesthesiology, 3. Radiologic Procedures; CT Scan; MRIAnesthetic Implications/Management, Anesthesia in Locations Outside the Operating Rooms

http://mycmecredit.com/ASA/ACEonline/printDiscussions.php

Page 55 of 108

Print ACE11A

5/25/14, 3:36 PM

QUESTION

54.

When performing a single injection, large volume (>15 mL) thoracic paravertebral block,
which of the following complications is MOST likely to occur?
A. Nausea and vomiting
B. Neuraxial spread of the solution
C. Hypotension
D. Pneumothorax

Comments on the incorrect answers


A.

Nausea and vomiting occurs in up to 17% of patients receiving a paravertebral block but is not the
most likely to occur.

C.

Hypotension occurs in approximately 4% of patients receiving a thoracic paravertebral block without


neuraxial spread and in only a portion of those who experience a neuraxial spread.

D.

Pneumothorax has been reported to occur in up to 0.5% of patients receiving a thoracic


paravertebral block.

Correct answer is B

Taxonomy
II.C. ADVANCED: Organ-based Basic and Clinical Sciences, 1. Central and Peripheral Nervous Systems

http://mycmecredit.com/ASA/ACEonline/printDiscussions.php

Page 56 of 108

Print ACE11A

5/25/14, 3:36 PM

QUESTION

55.

In managing perioperative opioid withdrawal in a patient with anxiety, muscle aches,


insomnia, runny nose, sweating, nausea, and vomiting, the BEST approach would be to
administer
A.

B.

C.

D.

regional anesthesia.

a mu receptor agonist.

N-methyl-D-aspartate (NMDA) receptor antagonists.

-antagonists.

Comments on the incorrect answers


A.

C.

D.

Regional anesthesia can be used to treat perioperative surgical pain, but it does not treat opioid
withdrawal.

Administration of NMDA receptor antagonists may be beneficial in perioperative pain management


but is ineffective in treating withdrawal symptoms.

Administration of -antagonists may be beneficial in perioperative pain management but is


ineffective in treating withdrawal symptoms.

Correct answer is B

Taxonomy
II.C. ADVANCED: Organ-based Basic and Clinical Sciences, 1. Central and Peripheral Nervous Systems

http://mycmecredit.com/ASA/ACEonline/printDiscussions.php

Page 57 of 108

Print ACE11A

5/25/14, 3:36 PM

QUESTION

56.

Which statement about carcinoid syndrome in a patient with a carcinoid tumor is MOST likely
true?
A. Over half of all patients with carcinoid tumor will develop carcinoid syndrome.
B. In patients with an isolated tumor, the risk of developing carcinoid syndrome is greater in those
patients with rectal tumors than those with ovarian tumors.
C. Less than half of all patients with carcinoid tumor will develop cardiac manifestations of carcinoid
syndrome.
D. Bronchoconstriction is the most common manifestation of carcinoid syndrome.

Comments on the incorrect answers


A.

Carcinoid syndrome occurs in only about 20% of patients with carcinoid tumors.

B.

In patients with carcinoid tumor of the small bowel, development of carcinoid syndrome is commonly
associated with the development of hepatic metastases. Isolated ovarian and intrathoracic tumors
cause carcinoid syndrome. Rectal tumors are rarely associated with development of carcinoid
syndrome.

D.

Although bronchospasm may occur, it is less common than flushing, diarrhea, and right-sided heart
disease that constitute carcinoid syndrome.

Correct answer is C

Taxonomy
II.C. ADVANCED: Organ-based Basic and Clinical Sciences, 7. Endocrine and Metabolic Systems

http://mycmecredit.com/ASA/ACEonline/printDiscussions.php

Page 58 of 108

Print ACE11A

5/25/14, 3:36 PM

QUESTION

57.

Which of the following is MOST likely to be an anesthetic management consideration in a


patient with systemic lupus erythematosus (SLE)?
A. Atlantoaxial subluxation
B. Obstructive pulmonary disease
C. Pituitary hyperactivity
D. Tracheomalacia

Comments on the incorrect answers


B.

Lupus patients have a restrictive pulmonary disease.

C.

Pituitary hyperactivity is not associated with SLE.

D.

Patients with SLE tend to acquire subglottic stenosis, not tracheomalacia.

Correct answer is A

Taxonomy
I.B. BASIC: Clinical Sciences: Anesthesia Procedures, Methods, and Techniques, 1. Evaluation of the
Patient and Preoperative Preparation

http://mycmecredit.com/ASA/ACEonline/printDiscussions.php

Page 59 of 108

Print ACE11A

5/25/14, 3:36 PM

QUESTION

58.

Which of the following is MOST likely to be present in a patient with LambertEaton


myasthenic syndrome (LEMS)?
A.

B.

C.

D.

Resistance to depolarizing neuromuscular blocking agents

Autonomic dysfunction

Resistance to nondepolarizing neuromuscular blocking agents

Decreased strength with repetitive activity

Comments on the incorrect answers


A.

C.

D.

Patients with LEMS have increased sensitivity to depolarizing neuromuscular blocking agents.

Patients with LEMS have increased sensitivity to nondepolarizing neuromuscular blocking agents.

Patients with LEMS exhibit increased strength with repetitive activity.

Correct answer is B

Taxonomy
II.C. ADVANCED: Organ-based Basic and Clinical Sciences, 8. Neuromuscular Diseases and Disorders:
Clinical Science

http://mycmecredit.com/ASA/ACEonline/printDiscussions.php

Page 60 of 108

Print ACE11A

5/25/14, 3:36 PM

QUESTION

59.

Which of the following statements about local anesthetics is MOST likely true?
A. Amino esters are eliminated primarily via hepatic transformation.
B. Local anesthetics with a low pKa have the longest duration of block.
C. Increased lipid solubility is associated with increased potency.
D. When applied to a peripheral nerve, the addition of epinephrine results in a shortened time to
onset of block.

Comments on the incorrect answers


A.

Amino esters are primarily hydrolyzed by plasma cholinesterase within the blood.

B.

Local anesthetics with the lowest pKa are associated with the most rapid onset but not a longer
duration of block.

D.

When applied to a peripheral nerve, the addition of epinephrine is not associated with a shortened
time to onset of block but is associated with increased block intensity, prolonged duration of block,
and decreased systemic absorption of local anesthetic.

Correct answer is C

Taxonomy
I.A. BASIC: Basic Sciences, 4. Pharmacology

http://mycmecredit.com/ASA/ACEonline/printDiscussions.php

Page 61 of 108

Print ACE11A

5/25/14, 3:36 PM

QUESTION

60.

(9B-53) A 44-year-old woman underwent insertion of a biventricular implantable cardioverter


defibrillator (ICD) for idiopathic cardiomyopathy. This was performed under monitored
anesthesia care with propofol and fentanyl. Vital signs at the end of the procedure included
heart rate 75 beats/min, blood pressure 105/72 mm Hg, SaO2 98% on 2L O2 via nasal cannula
(NC). One hour postoperatively, she is noted to be anxious and has the following vital signs:
sinus tachycardia at 130 beats/min, blood pressure 68/45 mm Hg, SaO2 89% on 2L O2 NC.
Which of the following would be the MOST appropriate immediate management step for this
patient?
A. Place a magnet over the ICD.
B. Obtain a transthoracic echocardiogram.
C. Perform coronary angiography.
D. Obtain a chest computed tomography (CT) scan.

Comments on the incorrect answers


A.

Placing a magnet on modern ICDs will disable the antitachycardia therapies and is not warranted at
this time.

C.

After ICD placement, a different diagnostic procedure should be performed prior to coronary
angiography to rule out cardiac tamponade.

D.

Chest CT is a sensitive test but is more time consuming than another option to rule out cardiac
tamponade.

Correct answer is B

Taxonomy
http://mycmecredit.com/ASA/ACEonline/printDiscussions.php

Page 62 of 108

Print ACE11A

5/25/14, 3:36 PM

II.C. ADVANCED: Organ-based Basic and Clinical Sciences, 3. Cardiovascular System

http://mycmecredit.com/ASA/ACEonline/printDiscussions.php

Page 63 of 108

Print ACE11A

5/25/14, 3:36 PM

QUESTION

61.

Compared to a regimen that involves opioids alone for the treatment of postoperative pain,
administration of nonsteroidal antiinflammatory drugs (NSAIDs) in conjunction with opioids
is MOST likely to result in which of the following?
A. Increased sedation
B. Reduced nausea
C. Reduced hepatic function
D. Increased hallucinations

Comments on the incorrect answers


A.

A regimen that involves the use of NSAIDs in conjunction with opioids is associated with a
significantly decreased risk of sedation compared with use of opioids alone.

C.

Perioperative administration of NSAIDs is not associated with hepatic dysfunction.

D.

Perioperative administration of NSAIDs is not associated with an increased rate of postoperative


hallucinations.

Correct answer is B

Taxonomy
I.B. BASIC: Clinical Sciences: Anesthesia Procedures, Methods, and Techniques, 7. Postoperative Period
II.D. ADVANCED: Clinical Subspecialties, 1. Painful Disease States

http://mycmecredit.com/ASA/ACEonline/printDiscussions.php

Page 64 of 108

Print ACE11A

5/25/14, 3:36 PM

QUESTION

62.

A patient is undergoing electroencephalographic (EEG) monitoring during a carotid


endarterectomy performed under general anesthesia. Which of the following EEG changes is
MOST consistent with severe cerebral ischemia?
A. Increased amplitude
B. Increased presence of fast frequencies
C. Preservation of amplitude
D. Loss of amplitude

Comments on the incorrect answers


A.

EEG amplitude is not increased in the presence of severe cerebral ischemia.

B.

Severe ischemia causes an increase in theta waves (slow frequency waves) on EEG.

C.

EEG amplitude is not maintained in the presence of severe cerebral ischemia.

Correct answer is D

Taxonomy
II.A. ADVANCED: Basic Sciences, 1. Physics, Monitoring, and Anesthesia Delivery Devices
II.C. ADVANCED: Organ-based Basic and Clinical Sciences, 1. Central and Peripheral Nervous Systems
II.C. ADVANCED: Organ-based Basic and Clinical Sciences, 3. Cardiovascular System

http://mycmecredit.com/ASA/ACEonline/printDiscussions.php

Page 65 of 108

Print ACE11A

5/25/14, 3:36 PM

QUESTION

63.

A transesophageal echocardiogram (TEE) is obtained in the transgastric mid short-axis view.


The left ventricle (LV) wall designated by the arrow is MOST likely supplied by which of the
following coronary arteries?
A. Left circumflex
B. Obtuse marginal
C. Left anterior descending
D. Right coronary

Comments on the incorrect answers


A.

The arrow shown in the TEE is pointing to the septal wall of the left ventricle. The circumflex artery
supplies the lateral and inferolateral or posterior walls of the left ventricle.

B.

The arrow shown in the TEE is pointing to the septal wall of the left ventricle. The obtuse marginal
artery, a branch of the circumflex artery, supplies the lateral and inferolateral walls of the left
ventricle.

D.

The arrow shown in the TEE is pointing to the septal wall of the left ventricle. The right coronary
artery supplies the right ventricle and inferior wall of the left ventricle.

Correct answer is C

Taxonomy
II.C. ADVANCED: Organ-based Basic and Clinical Sciences, 3. Cardiovascular System

http://mycmecredit.com/ASA/ACEonline/printDiscussions.php

Page 66 of 108

Print ACE11A

5/25/14, 3:36 PM

QUESTION

64.

Based on the American Society of Anesthesiologists (ASA) definitions, which of the following
characteristics BEST describes patients in a state of deep sedation?
A. No airway intervention is required.
B. Spontaneous ventilation is frequently inadequate.
C. Cardiovascular function is commonly impaired.
D. Responsiveness is only purposeful following repeated or painful stimulation.

Comments on the incorrect answers


A.

This describes a patient under moderate or lighter sedation.

B.

This describes a patient under general anesthesia.

C.

This does not describe a patient experiencing deep sedation; impaired cardiovascular function is an
indication of general anesthesia.

Correct answer is D

Taxonomy
I.B. BASIC: Clinical Sciences: Anesthesia Procedures, Methods, and Techniques, 4. Monitored Anesthesia
Care and Sedation: ASA Guidelines for Sedation, Sedation Guidelines for Non-Anesthesiologists

http://mycmecredit.com/ASA/ACEonline/printDiscussions.php

Page 67 of 108

Print ACE11A

5/25/14, 3:36 PM

QUESTION

65.

Which of the following is MOST likely included among the diagnostic criteria for obesity
hypoventilation syndrome (OHS)?
A.

B.

C.

D.

Central obesity

Awake hypercapnia (PaCO2 > 45 mm Hg)

Presence of obstructive sleep apnea (OSA)

Normal polysomnography findings

Comments on the incorrect answers


A.

C.

D.

Although the diagnosis of OHS requires a body mass index greater than 30 kg/m2, the specific
distribution of obesity is not a requirement for the diagnosis.

OSA is frequently found in patients with OHS, but it is not a requirement for the diagnosis.

Polysomnography findings are abnormal and include hypoventilation with nocturnal hypercapnia.

Correct answer is B
http://mycmecredit.com/ASA/ACEonline/printDiscussions.php

Page 68 of 108

Print ACE11A

5/25/14, 3:36 PM

Taxonomy
I.B. BASIC: Clinical Sciences: Anesthesia Procedures, Methods, and Techniques, 1. Evaluation of the
Patient and Preoperative Preparation
II.C. ADVANCED: Organ-based Basic and Clinical Sciences, 4. Gastrointestinal / Hepatic Systems

http://mycmecredit.com/ASA/ACEonline/printDiscussions.php

Page 69 of 108

Print ACE11A

5/25/14, 3:36 PM

QUESTION

66.

A patient with a carboxyhemoglobin concentration of 25% is MOST likely to have which of the
following?
A. Nausea
B. Coma
C. Cardiac dysrhythmias
D. Cherry-red colored blood

Comments on the incorrect answers


B.

Patients with carbon monoxide poisoning generally do not become comatose unless the
carboxyhemoglobin concentration is greater than 40%.

C.

Patients with carbon monoxide poisoning generally do not develop cardiac dysrhythmias unless the
carboxyhemoglobin concentration is greater than 40%.

D.

Patients with carbon monoxide poisoning generally do not develop cherry-red colored blood unless
the carboxyhemoglobin concentration is greater than 40%.

Correct answer is A

Taxonomy
II.C. ADVANCED: Organ-based Basic and Clinical Sciences, 1. Central and Peripheral Nervous Systems

http://mycmecredit.com/ASA/ACEonline/printDiscussions.php

Page 70 of 108

Print ACE11A

5/25/14, 3:36 PM

QUESTION

67.

Which of the following BEST describes the purpose of having isolated electrical power in the
operating room (OR)?
A. It minimizes the risk of microshock.
B. It eliminates the need to have equipment with a 3-prong grounded plug.
C. It minimizes the risk of macroshock.
D. It provides uninterruptible power to the OR.

Comments on the incorrect answers


A.

An isolated power supply does not provide protection against microshock.

B.

An isolated power supply does not eliminate the need to have 3-prong grounded plugs in the OR.

D.

An isolated power supply does not provide uninterruptible power to the OR.

Correct answer is C

Taxonomy
I.A. BASIC: Basic Sciences, 2. Physics, Monitoring, and Anesthesia Delivery Devices
II.A. ADVANCED: Basic Sciences, 1. Physics, Monitoring, and Anesthesia Delivery Devices

http://mycmecredit.com/ASA/ACEonline/printDiscussions.php

Page 71 of 108

Print ACE11A

5/25/14, 3:36 PM

QUESTION

68.

Which of the following is MOST likely to increase the risk of perioperative corneal abrasion?
A. Regional rather than general anesthesia
B. Case duration less than 90 minutes
C. Procedure involving the foot rather than the head and neck
D. Preexisting proptosis

Comments on the incorrect answers


A.

The risk of corneal abrasion is higher in patients undergoing general anesthesia.

B.

The risk of corneal abrasion is higher in patients undergoing longer procedures.

C.

The risk of corneal abrasion is higher in patients undergoing head and neck procedures.

Correct answer is D

Taxonomy
I.B. BASIC: Clinical Sciences: Anesthesia Procedures, Methods, and Techniques, 6. Complications
(Etiology, Prevention, Treatment)

http://mycmecredit.com/ASA/ACEonline/printDiscussions.php

Page 72 of 108

Print ACE11A

5/25/14, 3:36 PM

QUESTION

69.

You are caring for a patient who has suffered an aneurysmal subarachnoid hemorrhage
(SAH). Which statement about this patient is MOST likely true?
A. Hypothermia during surgery improves outcome.
B. Delayed cerebral ischemia, if it occurs, usually occurs 3 to 14 days after SAH.
C. Barbiturate coma should be routinely instituted.
D. Diltiazem is the only pharmacologic agent shown to improve outcome.

Comments on the incorrect answers


A.

Hypothermia during surgery does not improve outcome.

C.

Barbiturate coma may be indicated for the treatment of refractory status epilepticus but is not
routinely indicated for treatment of SAH.

D.

Nimodipine is the only pharmacologic agent shown to improve outcome.

Correct answer is B

Taxonomy
II.C. ADVANCED: Organ-based Basic and Clinical Sciences, 1. Central and Peripheral Nervous Systems

http://mycmecredit.com/ASA/ACEonline/printDiscussions.php

Page 73 of 108

Print ACE11A

5/25/14, 3:36 PM

QUESTION

70.

Which statement about a spontaneously breathing patient anesthetized with only an inhaled
anesthetic agent at 1.5 minimum alveolar concentration is MOST likely true?
A.

B.

C.

D.

All inhaled anesthetic agents cause an increase in tidal volume.

All inhaled anesthetic agents increase laryngeal irritant receptor sensitivity.

All inhaled anesthetic agents cause a decrease in respiratory rate.

A patient receiving desflurane will have a lower PaCO2 than a patient receiving sevoflurane.

Comments on the incorrect answers


A.

C.

D.

All inhaled anesthetic agents cause a decrease in tidal volume.

Isoflurane increases respiratory rate.

Desflurane produces a higher PaCO2 than sevoflurane.

Correct answer is B

Taxonomy
I.A. BASIC: Basic Sciences, 4. Pharmacology
II.A. ADVANCED: Basic Sciences, 2. Pharmacology

http://mycmecredit.com/ASA/ACEonline/printDiscussions.php

Page 74 of 108

Print ACE11A

5/25/14, 3:36 PM

QUESTION

71.

Which of the following is MOST likely to be removed in the pulmonary circulation?


A. Epinephrine
B. Dopamine
C. Norepinephrine
D. Isoproterenol

Comments on the incorrect answers


A.

Epinephrine is not eliminated by the pulmonary circulation.

B.

Dopamine is not eliminated by the pulmonary circulation.

D.

Isoproterenol is not eliminated by the pulmonary circulation.

Correct answer is C

Taxonomy
I.C. BASIC: Organ-based Basic and Clinical Sciences, 2. Respiratory System

http://mycmecredit.com/ASA/ACEonline/printDiscussions.php

Page 75 of 108

Print ACE11A

5/25/14, 3:36 PM

QUESTION

72.

Which of the following is the PRIMARY afferent nerve involved in the laryngeal closure reflex
during laryngospasm?
A. Glossopharyngeal
B. Superior laryngeal
C. Trigeminal
D. Recurrent laryngeal

Comments on the incorrect answers


A.

The glossopharyngeal nerve is the primary afferent nerve for stimuli in the oropharynx and
hypopharynx. While stimulus of the glossopharyngeal nerve may contribute to laryngospasm, it is not
the primary afferent nerve involved in the laryngeal closure reflex.

C.

The trigeminal nerve is the primary afferent nerve for stimuli in the nasopharynx. While stimulus of
the trigeminal nerve may contribute to laryngospasm, it is not the primary afferent nerve involved in
the laryngeal closure reflex.

D.

The recurrent laryngeal nerve is the primary efferent nerve for laryngeal closure during
laryngospasm.

Correct answer is B

Taxonomy
I.B. BASIC: Clinical Sciences: Anesthesia Procedures, Methods, and Techniques, 6. Complications
(Etiology, Prevention, Treatment)
I.C. BASIC: Organ-based Basic and Clinical Sciences, 2. Respiratory System

http://mycmecredit.com/ASA/ACEonline/printDiscussions.php

Page 76 of 108

Print ACE11A

5/25/14, 3:36 PM

QUESTION

73.

Which of the following is MOST likely to occur in a patient with severe hypophosphatemia
(serum phosphate level < 1.0 mg/dL)?
A. Muscle weakness
B. Seizures
C. Prolonged QT interval
D. Bronchospasm

Comments on the incorrect answers


B.

The central nervous system manifestations of hypophosphatemia include altered mental status, gait
disturbances, and paresthesias. Seizures occur in conjunction with hyperphosphatemia.

C.

The cardiovascular manifestations of hypophosphatemia include decreased myocardial contractility


and, if longstanding, cardiomyopathy. Prolonged QT interval occurs as a result of hypocalcemia,
which is caused by hyperphosphatemia.

D.

The primary ventilatory consequence of hypophosphatemia is muscle weakness, which may be


sufficiently severe as to result in acute respiratory failure. Bronchospasm occurs as a result of
hypocalcemia, which is caused by hyperphosphatemia.

Correct answer is A

Taxonomy
I.C. BASIC: Organ-based Basic and Clinical Sciences, 5. Renal and Urinary Systems/ Electrolyte Balance
II.C. ADVANCED: Organ-based Basic and Clinical Sciences, 5. Renal and Urinary Systems/ Electrolyte
Balance

http://mycmecredit.com/ASA/ACEonline/printDiscussions.php

Page 77 of 108

Print ACE11A

5/25/14, 3:36 PM

QUESTION

74.

Which of the following is MOST likely to cause development of acquired von Willebrand
syndrome (aVWS)?
A. Plasmapheresis
B. Severe aortic insufficiency
C. Hyperthyroidism
D. Presence of a nonpulsatile left ventricular assist device

Comments on the incorrect answers


A.

Plasmapheresis is one of the recommended forms of therapy for aVWS.

B.

Severe aortic stenosis, not aortic insufficiency, has been associated with aVWS (likely because of
the high shear stress to which the blood and major vasculature are exposed).

C.

Hypothyroidism, not hyperthyroidism, has been associated with decreased production and
decreased storage of von Willebrand factor, leading to aVWS.

Correct answer is D

Taxonomy
II.C. ADVANCED: Organ-based Basic and Clinical Sciences, 6. Hematologic System

http://mycmecredit.com/ASA/ACEonline/printDiscussions.php

Page 78 of 108

Print ACE11A

5/25/14, 3:36 PM

QUESTION

75.

Which of the following is MOST likely to be increased 30 minutes after a subject accustomed
to living at sea level rapidly ascends to an altitude of 8,000 feet?
A.

B.

C.

D.

Partial pressure of carbon dioxide in the alveolus (PaCO2)

Partial pressure of oxygen in arterial blood (PaO2)

Minute ventilation

Partial pressure of oxygen at which hemoglobin is 50% saturated (P50)

Comments on the incorrect answers


A.

B.

D.

Abrupt ascent to a higher altitude causes an increased ventilatory drive with an initial phase of
hyperventilation resulting in a decrease in the PaCO2.

Increased altitude is associated with decreased barometric pressure that results in decreased PaO2.

The acute respiratory alkalosis occurring as a result of ascent to a higher altitude results in a leftward
shift of the oxyhemoglobin dissociation curve that is reflected in a decreased P50.

Correct answer is C

Taxonomy
I.C. BASIC: Organ-based Basic and Clinical Sciences, 2. Respiratory System
II.B. ADVANCED: Clinical Sciences (Procedures, Methods, Techniques), 2. Special Techniques

http://mycmecredit.com/ASA/ACEonline/printDiscussions.php

Page 79 of 108

Print ACE11A

5/25/14, 3:36 PM

QUESTION

76.

Which of the following agents is MOST likely to produce bronchoconstriction in the parturient
with reactive airway disease?
A. Methylergonovine (Methergine)
B. Carboprost tromethamine (Hemabate)
C. Oxytocin (Pitocin)
D. Terbutaline (Brethine)

Comments on the incorrect answers


A.

Methylergonovine can precipitate hypertension rather than bronchoconstriction.

C.

Oxytocin is associated with hypotension, not bronchoconstriction, when administered in bolus doses.

D.

Terbutaline is not associated with bronchoconstriction.

Correct answer is B

Taxonomy
II.D. ADVANCED: Clinical Subspecialties, 3. Obstetric Anesthesia

http://mycmecredit.com/ASA/ACEonline/printDiscussions.php

Page 80 of 108

Print ACE11A

5/25/14, 3:36 PM

QUESTION

77.

A patient receiving chronic lithium therapy for treatment of bipolar disorder is MOST likely to
be at increased risk for developing which of the following?
A. Syndrome of inappropriate antidiuretic hormone secretion (SIADH)
B. Hypothyroidism
C. Peaked T waves on an electrocardiogram (ECG)
D. Leukopenia

Comments on the incorrect answers


A.

Up to 20% of patients on lithium therapy develop nephrogenic diabetes insipidus; they are not at risk
for developing SIADH.

C.

Patients receiving chronic lithium therapy commonly manifest flattened T waves on ECG; they are
not at increased risk for the development of peaked T waves.

D.

Patients on chronic lithium therapy have been reported to develop a leukocytosis with white blood
cell counts as high as 14,000/mm3 attributed solely to lithium.

Correct answer is B

Taxonomy
I.B. BASIC: Clinical Sciences: Anesthesia Procedures, Methods, and Techniques, 1. Evaluation of the
Patient and Preoperative Preparation

http://mycmecredit.com/ASA/ACEonline/printDiscussions.php

Page 81 of 108

Print ACE11A

5/25/14, 3:36 PM

QUESTION

78.

A 55-year-old alcoholic experiences arterial desaturation when standing that improves when
supine. Which process is MOST likely to be responsible for this phenomenon?
A. Chronic pulmonary emboli
B. Splenorenal shunting
C. Portopulmonary hypertension
D. Hepatopulmonary syndrome

Comments on the incorrect answers


A.

While chronic pulmonary emboli can lead to both chronic hypoxemia and pulmonary hypertension,
they are not the most likely cause of positional hypoxemia in this patient.

B.

Creation of a splenorenal shunt is employed to relieve portal hypertension; it does not explain the
positional hypoxia in this patient.

C.

Portopulmonary hypertension is defined as the presence of otherwise idiopathic pulmonary


hypertension in a patient with portal hypertension. It is not associated with orthodeoxia.

Correct answer is D

Taxonomy
I.B. BASIC: Clinical Sciences: Anesthesia Procedures, Methods, and Techniques, 1. Evaluation of the
Patient and Preoperative Preparation
II.C. ADVANCED: Organ-based Basic and Clinical Sciences, 4. Gastrointestinal / Hepatic Systems

http://mycmecredit.com/ASA/ACEonline/printDiscussions.php

Page 82 of 108

Print ACE11A

5/25/14, 3:36 PM

QUESTION

79.

Administration of a continuous infusion of fenoldopam at a rate of 0.1 g kg1 min1 is


MOST likely to result in a decrease in
A. urine output.
B. renal vascular resistance.
C. sodium excretion.
D. creatinine clearance.

Comments on the incorrect answers


A.

At a dose of 0.1 g kg1 min1, fenoldopam increases urine output.

C.

At a dose of 0.1 g kg1 min1, fenoldopam increases sodium excretion.

D.

At a dose of 0.1 g kg1 min1, fenoldopam increases creatinine clearance.

Correct answer is B

Taxonomy
I.C. BASIC: Organ-based Basic and Clinical Sciences, 5. Renal and Urinary Systems/ Electrolyte Balance
II.C. ADVANCED: Organ-based Basic and Clinical Sciences, 5. Renal and Urinary Systems/ Electrolyte
Balance

http://mycmecredit.com/ASA/ACEonline/printDiscussions.php

Page 83 of 108

Print ACE11A

5/25/14, 3:36 PM

QUESTION

80.

An otherwise healthy 3-month-old African-American male who was born at term gestation is
scheduled for an outpatient inguinal herniorrhaphy. A hemoglobin level of 9.9 g/dL was
obtained last week at a routine well child visit. The MOST appropriate next step is to
A. order a hemoglobin electrophoresis.
B. reschedule the procedure 1 week after initiation of iron therapy.
C. transfuse packed red blood cells.
D. proceed with surgery.

Comments on the incorrect answers


A.

A hemoglobin electrophoresis would not be expected to give additional information unless a


hemoglobinopathy is strongly suspected.

B.

The child does not need iron therapy, but if he did, it would take 2 weeks to start to have an effect.

C.

Transfusion of packed red blood cells is not indicated for a minor elective surgical procedure.

Correct answer is D

Taxonomy
II.D. ADVANCED: Clinical Subspecialties, 2. Pediatric Anesthesia

http://mycmecredit.com/ASA/ACEonline/printDiscussions.php

Page 84 of 108

Print ACE11A

5/25/14, 3:36 PM

QUESTION

81.

An otherwise healthy 10-month-old girl is undergoing an elective surgical procedure utilizing


a total intravenous anesthesia technique due to a family history of malignant hyperthermia.
After receiving a rapid bolus of fentanyl, the patient cannot be mask ventilated effectively. No
improvement in ventilation occurs after opening the patients airway with a jaw thrust or
placement of an oral airway. The MOST appropriate next step is to
A. insert a laryngeal mask airway (LMA).
B. administer albuterol.
C. insert a nasal airway.
D. administer rocuronium.

Comments on the incorrect answers


A.

The placement of a supraglottic airway device such as an LMA would not be expected to improve
ventilation since the most likely location of the obstruction is below the supraglottic structures.

B.

The administration of albuterol would not be expected to improve the ability to provide mask
ventilation.

C.

The placement of a nasal airway would not be expected to improve ventilation since the most likely
location of the obstruction is below the supraglottic structures.

Correct answer is D

Taxonomy
I.A. BASIC: Basic Sciences, 4. Pharmacology
II.D. ADVANCED: Clinical Subspecialties, 2. Pediatric Anesthesia

http://mycmecredit.com/ASA/ACEonline/printDiscussions.php

Page 85 of 108

Print ACE11A

5/25/14, 3:36 PM

QUESTION

82.

Assuming administration of the same dose of local anesthetic, injection at which of the
following sites will result in the LOWEST plasma concentration due to systemic absorption?
A. Caudal epidural
B. Lumbar epidural
C. Brachial plexus
D. Intercostal

Comments on the incorrect answers


A.

The injection of local anesthetic into the caudal space would not result in the lowest plasma
concentration of a local anesthetic.

B.

The injection of local anesthetic into the epidural space would not result in the lowest plasma

http://mycmecredit.com/ASA/ACEonline/printDiscussions.php

Page 86 of 108

Print ACE11A

5/25/14, 3:36 PM

concentration of a local anesthetic.


D.

The injection of local anesthetic into the intercostal space would not result in the lowest plasma
concentration of a local anesthetic.

Correct answer is C

Taxonomy
I.B. BASIC: Clinical Sciences: Anesthesia Procedures, Methods, and Techniques, 2. Regional Anesthesia
II.D. ADVANCED: Clinical Subspecialties, 2. Pediatric Anesthesia

http://mycmecredit.com/ASA/ACEonline/printDiscussions.php

Page 87 of 108

Print ACE11A

5/25/14, 3:36 PM

QUESTION

83.

A 9-year-old girl who received a heart transplant 5 years ago is scheduled for an elective
surgical procedure. Which of the following is the MOST appropriate anesthetic management
for this patient?
A.

B.

C.

D.

Restriction of intravenous fluids

Premedication with atropine

Avoidance of corticosteroids

Utilization of ST segment monitoring

Comments on the incorrect answers


A.

B.

C.

A patient with a transplanted heart is considered to be preload dependent due to a lack of cardiac
innervation in the donor heart; this results in the inability to produce tachycardia in response to
conditions such as hypovolemia. Therefore, avoidance of hypovolemia (and restriction of intravenous
fluids) is recommended.

Premedication with atropine is not routinely required for patients after cardiac transplantation.

It is common for corticosteroids to be administered chronically to patients after cardiac


transplantation. Strong consideration should be given to providing a stress dose of corticosteroids in
the preoperative period or soon after induction of anesthesia.

Correct answer is D

Taxonomy
II.C. ADVANCED: Organ-based Basic and Clinical Sciences, 3. Cardiovascular System

http://mycmecredit.com/ASA/ACEonline/printDiscussions.php

Page 88 of 108

Print ACE11A

5/25/14, 3:36 PM

QUESTION

84.

Which of the following is the MOST appropriate anesthetic management for an otherwise
healthy 5-week-old boy scheduled for a pyloromyotomy who has fasted for 8 hours?
A. Avoidance of succinylcholine
B. Administration of morphine
C. Performance of an inhalation induction
D. Gastric decompression immediately prior to induction of anesthesia

Comments on the incorrect answers


A.

Although the use of succinylcholine is controversial in pediatric patients, most experts recognize it is
an acceptable agent for infants undergoing pyloromyotomy.

B.

Administration of morphine is not recommended due to the increased risk for the development of
postoperative respiratory depression.

C.

Patients with pyloric stenosis are considered to be at increased risk for pulmonary aspiration of
gastric contents. Therefore, these patients are typically managed using a rapid-sequence induction
technique.

Correct answer is D

Taxonomy
II.D. ADVANCED: Clinical Subspecialties, 2. Pediatric Anesthesia

http://mycmecredit.com/ASA/ACEonline/printDiscussions.php

Page 89 of 108

Print ACE11A

5/25/14, 3:36 PM

QUESTION

85.

Administration of which of the following drugs is MOST likely to result in seizures in a patient
with renal insufficiency?
A. Meperidine
B. Morphine
C. Propofol
D. Dexmedetomidine

Comments on the incorrect answers


B.

Neither morphine nor its metabolites cause central nervous system excitation. Patients with renal
failure may develop life-threatening respiratory depression from high levels of morphine-6glucuronide.

C.

Propofol is a potent anticonvulsant; however, sedative doses have been reported to cause
myoclonus.

D.

Although dexmedetomidine has been shown to lower the seizure threshold in animal models, it has
proven to have minimal effects on children sedated for electroencephalograms or on patients
undergoing either electroconvulsive therapy or craniotomy for seizure focus resection.

Correct answer is A

Taxonomy
II.C. ADVANCED: Organ-based Basic and Clinical Sciences, 5. Renal and Urinary Systems/ Electrolyte
Balance

http://mycmecredit.com/ASA/ACEonline/printDiscussions.php

Page 90 of 108

Print ACE11A

5/25/14, 3:36 PM

QUESTION

86.

A 58-year-old homeless man with chronic alcoholism is admitted to the intensive care unit
because of pneumonia. He appears unkempt and emaciated. Intubation and mechanical
ventilation are required. A nasogastric tube is placed and enteral nutrition is initiated.
Subsequently, the patient develops generalized muscle weakness and dysrhythmias. Which
of the following conditions is MOST likely to have developed?
A. Hypoparathyroidism
B. Kwashiorkor
C. Hypervitaminosis D
D. Refeeding syndrome

Comments on the incorrect answers


A.

The clinical presentation is not consistent with hypoparathyroidism, which does not occur acutely
unless the parathyroids are removed during surgery.

B.

Kwashiorkor is characterized by marked muscle atrophy with normal or increased body fat and is
typically seen in malnourished individuals in developing countries.

C.

Hypervitaminosis D typically occurs after long-term administration of vitamin D rather than in the
clinical situation described.

Correct answer is D

Taxonomy
II.C. ADVANCED: Organ-based Basic and Clinical Sciences, 4. Gastrointestinal / Hepatic Systems

http://mycmecredit.com/ASA/ACEonline/printDiscussions.php

Page 91 of 108

Print ACE11A

5/25/14, 3:36 PM

QUESTION

87.

A 37-year-old woman is admitted to the intensive care unit with newly diagnosed thrombotic
thrombocytopenic purpura (TTP). She is somnolent but interactive. Her hemoglobin is 7.9
mg/dL; platelet count is 38 109/L. There is no evidence of bleeding. Which of the following
therapeutic interventions is MOST appropriate at this time?
A. Infusion of heparin
B. Administration of ciprofloxacin
C. Initiation of plasma exchange
D. Platelet transfusion

Comments on the incorrect answers


A.

Anticoagulants, including heparin, are not used to treat the microvascular thrombi of TTP, which are
mostly composed of platelets rather than fibrin.

B.

Ciprofloxacin has no role in the treatment of TTP.

D.

Platelet transfusion has been associated anecdotally with worsening of TTP symptoms, although
more recent investigations have cast doubt on this impression. Nonetheless, platelets should not be
administered to patients with TTP unless significant bleeding occurs.

Correct answer is C

Taxonomy
II.C. ADVANCED: Organ-based Basic and Clinical Sciences, 6. Hematologic System

http://mycmecredit.com/ASA/ACEonline/printDiscussions.php

Page 92 of 108

Print ACE11A

5/25/14, 3:36 PM

QUESTION

88.

Which statement about plasmapheresis is MOST likely true?


A. Administration of red blood cells is routinely required.
B. Central venous access is mandatory.
C. Heparin is removed by plasmapheresis.
D. Antibiotic concentration increases 3-fold after plasmapheresis.

Comments on the incorrect answers


A.

Red blood cell administration is not routinely required in patients undergoing plasmapheresis.

B.

Although central venous access is often used for plasmapheresis and is more efficient than
plasmapheresis through a peripheral IV, centrifugal plasmapheresis can be performed using largebore peripheral access.

D.

Most antibiotics are cleared, at least partially, by plasmapheresis. Consideration should be given to
re-dosing antibiotics after the procedure.

Correct answer is C

Taxonomy
II.C. ADVANCED: Organ-based Basic and Clinical Sciences, 6. Hematologic System

http://mycmecredit.com/ASA/ACEonline/printDiscussions.php

Page 93 of 108

Print ACE11A

5/25/14, 3:36 PM

QUESTION

89.

The partial pressure of oxygen (PO2) in which of the following locations is the GREATEST
stimulus for hypoxic pulmonary vasoconstriction?
A.

B.

C.

D.

Aortic sinus

Alveolus

Pulmonary artery

Bronchial artery

Comments on the incorrect answers


A.

C.

D.

The PO2 in the aortic sinus has an impact on hypoxic pulmonary vasoconstriction only to the degree
that it reflects the Po2 in the bronchial artery.

The PO2 in the pulmonary artery constitutes a relatively small portion of the stimulus for hypoxic
pulmonary vasoconstriction.

The PO2 in the bronchial arteries is a stimulus for vasoconstriction in the larger pulmonary arteries
but is not a major contributor to hypoxic pulmonary vasoconstriction.

Correct answer is B

Taxonomy
I.C. BASIC: Organ-based Basic and Clinical Sciences, 2. Respiratory System
II.C. ADVANCED: Organ-based Basic and Clinical Sciences, 2. Respiratory System

http://mycmecredit.com/ASA/ACEonline/printDiscussions.php

Page 94 of 108

Print ACE11A

5/25/14, 3:36 PM

QUESTION

90.

A 28-year-old man suffers a cervical spine injury during a motorcycle accident. Which of the
following manifestations is LEAST likely following an acute cervical spine injury?
A. Left ventricular dysfunction
B. Hypertension
C. Bradycardia
D. Pulmonary edema

http://mycmecredit.com/ASA/ACEonline/printDiscussions.php

Page 95 of 108

Print ACE11A

5/25/14, 3:36 PM

Comments on the incorrect answers


A.

Left ventricular dysfunction is associated with an acute cervical spine injury.

C.

Bradycardia is associated with an acute cervical spine injury.

D.

Pulmonary edema is associated with an acute cervical spine injury.

Correct answer is B

Taxonomy
I.C. BASIC: Organ-based Basic and Clinical Sciences, 1. Central and Peripheral Nervous Systems
II.C. ADVANCED: Organ-based Basic and Clinical Sciences, 1. Central and Peripheral Nervous Systems

http://mycmecredit.com/ASA/ACEonline/printDiscussions.php

Page 96 of 108

Print ACE11A

5/25/14, 3:36 PM

QUESTION

91.

Which of the following dysrhythmias is MOST likely in a patient with mitral valve prolapse
with severe mitral regurgitation?
A. Atrial fibrillation
B. Ventricular tachycardia
C. Ventricular fibrillation
D. Complete heart block

Comments on the incorrect answers


B.

Although ventricular tachycardia is associated with mitral valve prolapse, it is not the most common
dysrhythmia.

C.

Although ventricular fibrillation is associated with mitral valve prolapse, it is not the most common
dysrhythmia.

D.

Complete heart block is not typically associated with mitral valve prolapse.

Correct answer is A

Taxonomy
II.C. ADVANCED: Organ-based Basic and Clinical Sciences, 3. Cardiovascular System

http://mycmecredit.com/ASA/ACEonline/printDiscussions.php

Page 97 of 108

Print ACE11A

5/25/14, 3:36 PM

QUESTION

92.

Bolus administration of mannitol (1 g/kg) prior to the resection of a small frontal tumor is
MOST likely to result in a transient
A. increase in intracranial pressure.
B. decrease in central venous pressure.
C. increase in serum sodium concentration.
D. decrease in serum osmolality.

Comments on the incorrect answers


B.

There is a brief initial increase in central venous pressure during mannitol administration, followed by
a sustained decrease.

C.

Serum sodium concentration remains unchanged or decreases during mannitol administration due to
the osmotic expansion of the extracellular volume.

D.

Serum osmolality remains unchanged or increases due to mannitol administration.

Correct answer is A

Taxonomy
II.C. ADVANCED: Organ-based Basic and Clinical Sciences, 1. Central and Peripheral Nervous Systems

http://mycmecredit.com/ASA/ACEonline/printDiscussions.php

Page 98 of 108

Print ACE11A

5/25/14, 3:36 PM

QUESTION

93.

DIRECTIONS FOR ITEMS 93 AND 94: These items refer to the diagnosis, treatment, or
management of a single patient.
A 32-year-old woman has an electrocardiogram (ECG) in the preoperative clinic prior to a
knee arthroscopy: These findings are MOST consistent with which of the following?
A. Atrial flutter
B. Wolff-Parkinson-White syndrome
C. Second-degree heart block
D. Right bundle branch block

Comments on the incorrect answers


A.

ECG findings of atrial flutter include multiple p waves between each QRS complex.

C.

ECG findings that would indicate a second-degree heart block include a dropped or missing QRS
following a p wave.

D.

ECG findings of a right bundle branch block include an rSR pattern in V1 and a slurred S wave in V6.

Correct answer is B

Taxonomy
II.C. ADVANCED: Organ-based Basic and Clinical Sciences, 3. Cardiovascular System

http://mycmecredit.com/ASA/ACEonline/printDiscussions.php

Page 99 of 108

Print ACE11A

5/25/14, 3:36 PM

QUESTION

94.

Following the procedure, the patient is extubated and transported to the recovery room.
When reconnected to the monitors, her heart rate was 180 beats/min with the following
rhythm strip: Which of the following pharmacologic agents would be the MOST appropriate in
treating this condition?
A. Digoxin
B. Esmolol
C. Lidocaine
D. Nicardipine

Comments on the incorrect answers


A.

Digoxin inhibits the normal conduction pathway through the atrioventricular node and should be
avoided in patients with Wolff-Parkinson-White syndrome.

B.

Beta-receptor blocking agents inhibit the normal conduction pathway through the atrioventricular
node and should be avoided in patients with Wolff-Parkinson-White syndrome.

D.

Calcium channel blockers inhibit the normal conduction pathway through the atrioventricular node
and should be avoided in patients with Wolff-Parkinson-White syndrome.

Correct answer is C

Taxonomy
II.C. ADVANCED: Organ-based Basic and Clinical Sciences, 3. Cardiovascular System

http://mycmecredit.com/ASA/ACEonline/printDiscussions.php

Page 100 of 108

Print ACE11A

5/25/14, 3:36 PM

QUESTION

95.

DIRECTIONS FOR ITEMS 95 AND 96: These items refer to the diagnosis, treatment, or
management of a single patient.
A 33-year-old woman at 28 weeks gestation has been diagnosed with Guillain-Barr
syndrome after developing lower extremity weakness that has now progressed to involve the
upper extremities and intercostal muscles. She is complaining of difficulty breathing and her
vital capacity is 12 mL/kg. Recommended management options for this parturient would
include all of the following EXCEPT
A. plasmapheresis.
B. corticosteroid therapy.
C. intravenous immunoglobulin.
D. mechanical support of ventilation.

Comments on the incorrect answers


A.

Plasmapheresis is an effective treatment for Guillain-Barr syndrome and is not contraindicated in


pregnancy.

C.

Intravenous immunoglobulin is an effective treatment for Guillain-Barr syndrome and is not


contraindicated in pregnancy.

D.

Mechanical support of ventilation may be required when the patients vital capacity is less than 15
mL/kg.

Correct answer is B

Taxonomy
II.C. ADVANCED: Organ-based Basic and Clinical Sciences, 8. Neuromuscular Diseases and Disorders:
Clinical Science
II.D. ADVANCED: Clinical Subspecialties, 3. Obstetric Anesthesia

http://mycmecredit.com/ASA/ACEonline/printDiscussions.php

Page 101 of 108

Print ACE11A

5/25/14, 3:36 PM

QUESTION

96.

The patient now presents to the labor and delivery unit at 39 weeks gestation for a repeat
cesarean delivery. She reports that she has been able to walk for at least 4 weeks and her
residual weakness is decreasing. Which statement about the management of this patient is
MOST likely true?
A. Intraoperative hypotension should be treated with ephedrine.
B. Combined spinal-epidural (CSE) anesthesia is contraindicated.
C. Succinylcholine should be avoided.
D. If a nondepolarizing neuromuscular blocking drug is administered, the dose should be increased.

Comments on the incorrect answers


http://mycmecredit.com/ASA/ACEonline/printDiscussions.php

Page 102 of 108

Print ACE11A

5/25/14, 3:36 PM

A.

If treatment of neuraxial anesthesiainduced hypotension is required, the use of a direct-acting


adrenergic drug is generally recommended. Patients with Guillain-Barr syndrome may manifest an
exaggerated response to an indirect-acting sympathomimetic drug (eg, ephedrine) due to upregulation of the postsynaptic receptors.

B.

CSE anesthesia is not contraindicated. There have been several reports of successful CSE and
epidural anesthesia in parturients with Guillain-Barr syndrome.

D.

Patients with Guillain-Barr syndrome may have an increased sensitivity to nondepolarizing


neuromuscular blocking drugs. Therefore, the dose should be decreased, not increased.

Correct answer is C

Taxonomy
II.C. ADVANCED: Organ-based Basic and Clinical Sciences, 8. Neuromuscular Diseases and Disorders:
Clinical Science
II.D. ADVANCED: Clinical Subspecialties, 3. Obstetric Anesthesia

http://mycmecredit.com/ASA/ACEonline/printDiscussions.php

Page 103 of 108

Print ACE11A

5/25/14, 3:36 PM

QUESTION

97.

DIRECTIONS FOR ITEMS 97 AND 98: These items refer to the diagnosis, treatment, or
management of a single patient.
A patient in the emergency department has been exposed to an unknown chemical as a result
of an industrial accident. The patient has no obvious trauma but is confused, sweaty,
bradycardic, wheezing, and having difficulty managing oral secretions. Exposure to which of
the following agents has MOST likely caused these findings?
A. Cyanide
B. Organophosphates
C. Phosgene
D. Chlorine

Comments on the incorrect answers


A.

Cyanide exposure causes altered mental status, hyperventilation, hypertension, and tachycardia,
which may progress to seizures or coma with cardiovascular collapse and respiratory arrest.

C.

Phosgene exposure initially produces coughing, choking, chest tightness, and eye irritation.

D.

Chlorine exposure results in coughing, nausea, pulmonary edema, and burning in the nose, throat,
and mouth.

Correct answer is B

Taxonomy
II.C. ADVANCED: Organ-based Basic and Clinical Sciences, 1. Central and Peripheral Nervous Systems
II.D. ADVANCED: Clinical Subspecialties, 12. Critical Care

http://mycmecredit.com/ASA/ACEonline/printDiscussions.php

Page 104 of 108

Print ACE11A

5/25/14, 3:36 PM

QUESTION

98.

Which of the following is MOST likely to occur in this patient following administration of an
intubating dose of succinylcholine?
A. Increased duration of neuromuscular blockade
B. Hyperkalemic arrest
C. Ventricular tachycardia
D. Resistance to the effects of succinylcholine

Comments on the incorrect answers


B.

Hyperkalemic arrest is not a known complication of administering succinylcholine to a patient with


organophosphate poisoning.

C.

Ventricular tachycardia is not a known complication of administering succinylcholine to a patient with


organophosphate poisoning.

D.

Increased resistance to succinylcholine is seen in patients with myasthenia gravis, not with
organophosphate poisoning.

Correct answer is A

Taxonomy
I.A. BASIC: Basic Sciences, 4. Pharmacology
II.D. ADVANCED: Clinical Subspecialties, 12. Critical Care

http://mycmecredit.com/ASA/ACEonline/printDiscussions.php

Page 105 of 108

Print ACE11A

5/25/14, 3:36 PM

QUESTION

99.

DIRECTIONS FOR ITEMS 99 AND 100: These items refer to the diagnosis, treatment, or
management of a single patient.
A patient with recently diagnosed hyperthyroidism is in the postanesthesia care unit
following an emergency appendectomy 2 hours ago. The patient is febrile (41C) and
tachycardic (160 beats/min). Which of the following manifestations is MOST consistent with
the diagnosis of thyroid storm?
A. Hyponatremia
B. Cardiac tamponade
C. Hypovolemia
D. Decreased ventilatory response to hypoxemia

Comments on the incorrect answers


A.

Hyponatremia is a common finding in patients with myxedema coma. Serum sodium is generally
normal in patients with hyperthyroidism and thyroid storm.

B.

Cardiac tamponade due to accumulation of mucopolysaccharide-containing fluid in the pericardium


occurs with myxedema coma. There is no association between thyroid storm and cardiac
tamponade.

D.

Decreased ventilatory response to both hypoxemia and hypercapnia occurs in conjunction with
myxedema coma. Neither is reported to occur with thyroid storm.

Correct answer is C

Taxonomy
I.C. BASIC: Organ-based Basic and Clinical Sciences, 7. Endocrine and Metabolic Systems
II.C. ADVANCED: Organ-based Basic and Clinical Sciences, 7. Endocrine and Metabolic Systems

http://mycmecredit.com/ASA/ACEonline/printDiscussions.php

Page 106 of 108

Print ACE11A

5/25/14, 3:36 PM

QUESTION

100. Once the diagnosis of thyroid storm is made, which of the following treatments is MOST
likely to be contraindicated?
A. Aspirin
B. Hydrocortisone
C. Propranolol
D. Iodine
Comments on the incorrect answers
B.

Corticosteroids are indicated because of the potential for adrenal insufficiency that may occur in
conjunction with the increased metabolic demands and rapid turnover of cortisol due to thyroid
storm. Steroids may also be indicated if there is an autoimmune element to the disease.

C.

Beta-blockers not only decrease heart rate (and the associated increased myocardial oxygen
demand) but also result in some improvement in central nervous system manifestations. Propranolol
has a theoretical benefit over other beta-blockers because it inhibits conversion of T4 to T3.

D.

When administered after antithyroid drugs such as thionamides (eg, propylthiouracil), iodine
produces rapid decreases in serum levels of T4. If administered prior to antithyroid therapy, iodine
will enhance the synthesis of thyroid hormones, leading to increased stores in the gland.

Correct answer is A

Taxonomy
I.C. BASIC: Organ-based Basic and Clinical Sciences, 7. Endocrine and Metabolic Systems
II.C. ADVANCED: Organ-based Basic and Clinical Sciences, 7. Endocrine and Metabolic Systems

http://mycmecredit.com/ASA/ACEonline/printDiscussions.php

Page 107 of 108

Print ACE11A

http://mycmecredit.com/ASA/ACEonline/printDiscussions.php

5/25/14, 3:36 PM

Page 108 of 108

Vous aimerez peut-être aussi